Está en la página 1de 111

1

Xenopsylla cheopis is the vector responsible for which infectious disease?


1

Human Monocytic Ehrlichiosis
2

Human Granulocytic Ehrlichiosis
3

Scrub typhus
4

Epidemic typhus
5

Endemic typhus
Q/Q(M)-474269 Report a Problem
Xenopsylla cheopis is the vector responsible for which infectious disease?
5

Endemic typhus
Xenopsylla cheopis (rat flea) is the vector of endemic typhus, which is caused by R. typhi.
Q/Q(M)-474269 Report a Problem
Butcher's warts are caused by which human papilloma virus (HPV) type?
1

7
2

2
3

3
4

5
5

13
Q/Q(M)-474297 Report a Problem
Butcher's warts are caused by which human papilloma virus (HPV) type?
1

7
Butcher's warts are caused by HPV type 7.
Q/Q(M)-474297 Report a Problem
Tricomycosis axillaris is caused by:
1

Burkholderia mallei
2

Proteus species
3

Corynebacterium tenuis
4

Micrococcus sedentarius
5

Corynebacterium minitissimum
Q/Q(M)-474539 Report a Problem
Tricomycosis axillaris is caused by:
3

Corynebacterium tenuis
Corynebacterium tenuis is the causative organism associated with most cases. Although up to 33% of
adults have colonization by this bacterium in the inguinal or axillary regions, factors such as
hyperhidrosis predispose to more extensive growth and resultant clinical manifestations.
Q/Q(M)-474539 Report a Problem
A 43 year-old man with uncontrolled HIV disease presents with yellowish, red papules, draining sinuses
and ulcers perianally and periurethrally. Which histopathological or immunohistochemical stain may aid
in diagnosis?
1

CD3
2

CD20
2

3

Von Gieson (EVG)
4

Von Kossa
5

Steiner
Q/Q(M)-477607 Report a Problem
A 43 year-old man with uncontrolled HIV disease presents with yellowish, red papules, draining sinuses
and ulcers perianally and periurethrally. Which histopathological or immunohistochemical stain may aid
in diagnosis?
4

Von Kossa
The most likely diagnosis is malacoplakia. Malacoplakia, an infection usually attributed to S. aureus, P.
aeruginosa, or E. coli is characterized histopathologically by Michelis-Gutmann bodies. These are
foamy eosinophilic macrophages containing calcified, laminated, intracytoplasmic bodies. They can be
highlighted by a calcium stain such as von Kossa. CD3 and CD20 are lymphocyte markers, von Gieson
is an elastic tissue stain, and Steiner stain is used to highlight spirochetes. These would not aid in the
diagnosis of malacoplakia.
Q/Q(M)-477607 Report a Problem
Which of the following is true regarding the Gardasil vaccine?
1

It is a live vaccine
2

It is FDA approved for females aged 18 to 25
3

It protects against HPV 16 and 18 only
4

It can be administered regardless of history of abnormal pap smear
5

It does not protect against genital warts
Q/Q(M)-482490 Report a Problem
Which of the following is true regarding the Gardasil vaccine?
4

It can be administered regardless of history of abnormal pap smear
Gardasil is a quadrivalent vaccine containing HPV types 6, 11, 16 and 18. It is approved for females
ages 9 to 26. Neither HPV testing nor pap smears are necessary before vaccine administration. It is
recommended for the prevention of cervical, vaginal, and vulvar cancers, as well as for genital warts.
Q/Q(M)-482490 Report a Problem
Streptococcus Iniae has been shown to cause:
1

Perianal dermatitis in neonates
2

Necrotizing fasciitis
3

Bullous impetigo
4

Hand cellulitis in fish handlers
5

Perineal erysipelas in postpartum women
Q/Q(M)-477424 Report a Problem
Streptococcus Iniae has been shown to cause:
4

Hand cellulitis in fish handlers
Streptococcus iniae has been demonstrated to cause hand cellulitis from puncture wounds sustained
form the dorsal fin, fish bone or knife of usually a tilapia. Treatment with PCN is curative. Group A
Streptococci are the most common cause of perianal dermatitis. Many different bacteria have been
implicated in necrotizing fasciitis. Bullous impetigo is most frequently caused by phage type 71 S.
aureus or a related group 2 phage type. Group B streptococcus is most often responsible for perineal
3

erysipelas in postpartum women.
Q/Q(M)-477424 Report a Problem
The causative organism for Brucellosis is which of the following:
1

Gram positive rod
2

Gram positive cocci
3

Gram negative rod
4

Gram negative cocci
5

Acid fast bacilli
Q/Q(M)-477610 Report a Problem
The causative organism for Brucellosis is which of the following:
3

Gram negative rod
Brucella species which cause Brucellosis are gram negative rods. Brucellosis, aka undulant fever, is
characterized by an acute febrile illness with headache and joint pain. CNS and cardiac manifestations
can also occur. It is acquired by contact with infected animals or contaminated dairy products.
Treatment is with doxycycline and rifampin.
Q/Q(M)-477610 Report a Problem
What is the vector for this nematodal infection?
1

Water flea (Cyclops)
2

Tsetse fly (Glossine morsitans)
3

Reduviid bug
4

Mango fly (Chrysops)
5

Black fly (Simulium)
Q/Q(M)-476850 Report a Problem
What is the vector for this nematodal infection?
4

Mango fly (Chrysops)
Loiasis is caused by the nematode loa loa. It is transmitted by the mango fly (Chrysops dimidia or
CHrysops silacea). It may manifest with painful, localized subcutaneous nonpitting edema called calabar
swellings. Conjunctival migrations are also common.
Q/Q(M)-476850 Report a Problem
A patient from South America develops a recurrent fever after living in crowded conditions with
multiple roommates having known human body louse infestation. A diagnosis of louse-born Relapsing
Fever is made. Which of the following is the implicated organism?
1

Borrelia recurrentis
2

Borrelia duttonii
3

Borrelia hermsii
4

Borrelia burgdorferi
5

Streptobacillus moniliformis
Q/Q(M)-482686 Report a Problem
A patient from South America develops a recurrent fever after living in crowded conditions with
multiple roommates having known human body louse infestation. A diagnosis of louse-born Relapsing
Fever is made. Which of the following is the implicated organism?
4

1

Borrelia recurrentis
Louse-born relapsing fever is caused by Borrelia recurrentis. It is seen in Africa and South America, and
is spread by the human body louse (Pediculus humanus). Symptoms include paroxysmal fevers,
headache, myalgias, and petechial macules on trunk and extremities. Borrelia burgdorferi causes Lyme
disease; Borrelia duttonii and Borrelia hermsii cause tick-born Relapsing Fever; Streptobacillus
moniliformis causes Rat-bite fever.
Q/Q(M)-482686 Report a Problem
Pediculus humanus corporis can transmit:
1

Murine typhus
2

Epidemic typhus
3

Scrub typhus
4

Endemic typhus
5

Oriental typhus
Q/Q(M)-479607 Report a Problem
Pediculus humanus corporis can transmit:
2

Epidemic typhus
The body louse or Pediculus humanus corporis transmits Rickettsia prowasekii the organism responsible
for epidemic typhus. Endemic typhus or murine typhus is caused by R. typhi that is transmited by the rat
flea/Xenopsylla cheopis. Scrub typhus is caused by R. tsutsugamushi and is transmitted by chiggers or
trombiculid mite larvae.
Q/Q(M)-479607 Report a Problem
A 10 year-old with a 3-day history of fever and headache presents to the clinic with multiple
erythematous papulopustules on his extremities, many of which have central eschars. The most likely
diagnosis is:
1

Lyme disease
2

Rickettsialpox
3

Erlichiosis
4

Rocky mountain spotted fever
5

Meningococcemia
Q/Q(M)-477181 Report a Problem
A 10 year-old with a 3-day history of fever and headache presents to the clinic with multiple
erythematous papulopustules on his extremities, many of which have central eschars. The most likely
diagnosis is:
2

Rickettsialpox
Rickettsialpox is an acute febrile illness caused by the bacteria Rickettsia akari, a member of the spotted
fever group. R. akari is transmitted by the house mouse mite, Liponyssus sangineus. Clinically, patients
have an eschar at the inoculation site with a subsequent papulovesicular rash, fevers, headache, chills,
diaphoresis, myalgia, and anorexia.
Q/Q(M)-477181 Report a Problem
A 4 year-old boy is diagnosed with Staphylococcal Scalded-Skin Syndrome. Vesiculation in this disease
is secondary to exfoliative toxins binding what target protein?
1

Plectin
2

Bullous pemphigoid antigen 2
5

3

Bullous Pemphigoid antigen 1
4

Desmoglein 3
5

Desmoglein 1
Q/Q(M)-477602 Report a Problem
A 4 year-old boy is diagnosed with Staphylococcal Scalded-Skin Syndrome. Vesiculation in this disease
is secondary to exfoliative toxins binding what target protein?
5

Desmoglein 1
Staphylococcus Scalded-Skin Syndrome (SSSS) is caused by Staphylococcus exfoliative toxins A and B
binding desmoglein 1. This results in the intraepidermal split at the granular layer seen in this illness.
Targeting of desmoglein 3 would result in a suprabasilar split. Targeting of plectin, BPAG1, or BPAG2
would result in a subepidermal split. This is not seen in SSSS.
Q/Q(M)-477602 Report a Problem
Most common causative organism of the attached picture is
1

Corynebacterium minutissimum
2

Corynebacterium tenuis
3

Micrococcus sedentarius
4

Staphylococcus
5

Streptococcus
Q/Q(M)-482123 Report a Problem
Most common causative organism of the attached picture is
3

Micrococcus sedentarius
The image shows Pitted keratolysis. It is a non-inflammatory bacterial infection caused by
microcolonies of Micrococcus sedentarius or Corynebacterium species that produce a specific
proteolytic enzyme which digests the stratum corneum. It consists of small crater-like depressions in the
stratum corneum, and usually appear on the weight-bearing regions of the soles of the feet, but can occur
in other non weight-bearing areas as shown in image. This was confirmed with shave biopsy of
epidermis that is step-sectioned and stained with methenamine silver as this is found to be more helpful
than a punch biopsy. The disease usually goes unnoticed by patients and is most often an incidental
finding. Hyperhidrosis and malodor of the skin are very common findings. Micrococcus sedentarius is
the causative agent for erythrasma, and Corynebacterium tenius causes trichomycosis Axillaris.
Q/Q(M)-482123 Report a Problem

A 24 year old male presents with a high fever, arthralgias, and a rash characterized by nonspecific
erythematous macules in a generalized distribution. He was recently cleaning his family\'s barn, and was
bitten more than once by rats and mice. What is the treatment of choice for this infection?
6

1

Penicillin
2

Erythromycin
3

Clindamycin
4

Ciprofloxacin
5

Amikacin
Q/Q(M)-482688 Report a Problem
A 24 year old male presents with a high fever, arthralgias, and a rash characterized by nonspecific
erythematous macules in a generalized distribution. He was recently cleaning his family\'s barn, and was
bitten more than once by rats and mice. What is the treatment of choice for this infection?
1

Penicillin
Rat-bite fever is also called Haverhill Fever. It is caused by Streptobacillus moniliformis and is acquired
from infected rodents or by touching or eating contaminated food. The signs and symptoms are
nonspecific, but the diagnosis can be suspected based on history. Treatment is with penicillin.
Q/Q(M)-482688 Report a Problem
Which porphyrin is responsible for the fluorescence under Wood's lamp examination of erythrasma?
1

Porphobilinogen
2

Protoporphyrin IX
3

Coproporphyrin III
4

Uroporphobilinogen
5

Coproporphyrinogen
Q/Q(M)-476510 Report a Problem

Which porphyrin is responsible for the fluorescence under Wood's lamp examination of erythrasma?
3

Coproporphyrin III
Erythrasma is caused by corynebacterium minutissimum. The presence of coprophyrinogen III created
by the bacteria is responsible for the characteristic coral-red fluorescence under Wood's lamp.
Q/Q(M)-476510 Report a Problem

A 62 year-old man presents with tinnitus, facial paralysis, and vesicles of the external ear. What is the
most likely diagnosis?
1

Primary Herpes Simplex infection
2

Primary Varicella Infection
3

Herpes Zoster Infection
4

Cytomegalovirus infection
5

Coxsackievirus A16 infection
Q/Q(M)-478210 Report a Problem

A 62 year-old man presents with tinnitus, facial paralysis, and vesicles of the external ear. What is the
most likely diagnosis?
7

3

Herpes Zoster Infection
This patient has Ramsay Hunt Syndrome, herpes zoster infection of the geniculate ganglion. Vesicles
can be seen on the tympanic membrane and the external ear. Symptoms include ipsilateral facial
paralysis, tinnitus, or other auditory symptoms. Treatment is with acyclovir and systemic corticosteroids.
Q/Q(M)-478210 Report a Problem

A 59-year-old male returns from traveling abroad where he ate many fish. Since then he complaints of
recurring migratory 1 cm/day erythematous urticarial plaques. Biopsy shows eosinophilic panniculitis.
What is the most likely condition?
1

Drancunculiasis
2

Gnathostomiasis
3

Enterobiasis
4

Hookworm
5

Larva currents
Q/Q(M)-482310 Report a Problem

A 59-year-old male returns from traveling abroad where he ate many fish. Since then he complaints of
recurring migratory 1 cm/day erythematous urticarial plaques. Biopsy shows eosinophilic panniculitis.
What is the most likely condition?
2

Gnathostomiasis
This patient has gnathostomiasis caused by Gnathostoma dolorosi or spinigerum. It is commonly
acquired from freshwater fish. It manifests with migratory intermittent erythematous urticarial plaques
that recur every 2-6 weeks. The plaques move about 1 cm/day. Histology shows eosinophilic
panniculitis. Treatment is surgical removal or albendazole.
Q/Q(M)-482310 Report a Problem

All of the following statements are true of Bacillus anthracis infection except:
1

Spores remain stable for decades
2

20% mortality rate in untreated cutaneous infections
3

Inhalation, GI and cutaneous forms exist
4

Ciprofloxacin and doxycycline are first line treatments
5

All statements are true
Q/Q(M)-477209 Report a Problem
All of the following statements are true of Bacillus anthracis infection except:
5

All statements are true
Anthrax can present in three different clinical forms: cutaneous, inhalational, and gastrointestinal. The
cutaneous form appears as a painless vesicle that later forms an eschar (malignant pustule). The
cutaneous form has a mortality rate of less than 1% if treated an up to 20% if untreated.
Q/Q(M)-477209 Report a Problem
A 56 year old homeless man has many waxy concretions on his axilla and groin hairs. Which of the
following organisms is the likely cause?
8

1

Corynebacterium afermentans
2

Corynebacterium tenius
3

Corynebacterium diptheriae
4

Micrococcus sedentarius
5

Corynebacterium minutissimum
Q/Q(M)-480170 Report a Problem

A 56 year old homeless man has many waxy concretions on his axilla and groin hairs. Which of the
following organisms is the likely cause?
2

Corynebacterium tenius
This condition described is trichomycosis axillaris. Despite the mycosis in the name, it is not fungal
in etiology, but caused by C. tenius. Treatment is topical erythromycin or simply shaving the infected
hairs. The other listed organisms do not cause this type of infection.
Q/Q(M)-480170 Report a Problem

What is the most common presentation of mucous membrane involvement of EBV infection?
1

Buccal ulcers with superficial slough
2

Erosions of the gingivae
3

Plication of the tongue
4

Palatal petechiae
5

Oral hairy leukoplakia
Q/Q(M)-482590 Report a Problem
What is the most common presentation of mucous membrane involvement of EBV infection?
4

Palatal petechiae
The most common presentation of intra-oral involvement of EBV infection is pinpoint petechiae of the
junction of the hard and soft palate. The associated eponym is Forchheimer spots. Forchheimer spots are
not pathognomonic for EBV infection, as they can be seen in measles, rubella, and scarlet fever. While
oral hairy leukoplakia is associated with EBV infection, it is not the most common mucous membrane
sign.
Q/Q(M)-482590 Report a Problem

The treatment of choice of cat-scratch disease is:
1

Erythromycin
2

Ceftriaxone
3

Trimethoprim-sulfamethoxazole
4

Itraconazole
5

Mebendazole
Q/Q(M)-477121 Report a Problem
The treatment of choice of cat-scratch disease is:
1

Erythromycin
9

Cat-scratch disease is caused by Bartonella henselae, which is spread among cats by fleas. The hallmark
of the disease is unilateral and regional lymphadenopathy. The treatment of choice for cat-scratch
disease is erythromycin.
Q/Q(M)-477121 Report a Problem

A homeless patient with a history of diabetes and alcoholism presents with chronic suppurative nodules
with granular bodies on light microscopy. Culture rapidly grows Staphylococcus aureus colonies. What
is the clinical term describing this type of infectious lesion?
1

Botryomycosis
2

Scrofuloderma
3

Rhinoscleroma
4

Ecthyma gangrenosum
5

Erysipeloid
Q/Q(M)-482687 Report a Problem
A homeless patient with a history of diabetes and alcoholism presents with chronic suppurative nodules
with granular bodies on light microscopy. Culture rapidly grows Staphylococcus aureus colonies. What
is the clinical term describing this type of infectious lesion?
1

Botryomycosis
Chronic purulent nodules with granular bodies seen in patients with immunosuppresion, diabetes, and
alcoholism is called botryomycosis. Causative organisms include S. aureus, P. aeruginosa, E. coli, and
Proteus species. Scrofuloderma is cutaneous extension of underlying Mycobacteria tuberculosis
infection. Rhinoscleroma is caused by Klebsiella pneumoniae rhinoscleromatis. Ecthyma gangrenosum
is typically seen following Pseudomonas septicemia. Erysipeloid is caused by Erysipelothrix
rhusiopathiae.
Q/Q(M)-482687 Report a Problem

Which of the following HPV types causes a ridged wart?
1

HPV 1
2

HPV 7
3

HPV 11
4

HPV 18
5

HPV 60
Q/Q(M)-482317 Report a Problem
Which of the following HPV types causes a ridged wart?
5

HPV 60
HPV 60 infection leads to a particular type of plantar wart called the ridged wart. HPV 1 leads to plantar
warts and myrmecia. HPV 7 causes butcher's warts. HPV 11 along with 6 causes anogenital condyloma
and giant condyloma of Bucke and Lowenstein (verrucous carcinoma). HPV 18 causes anogenital
dysplasia and neoplasms.
Q/Q(M)-482317 Report a Problem

Calabar swellings are a characteristic feature of which disease?
10

1

Onchocerciasis
2

Eumycetoma
3

Tertiary Syphilis
4

Carrion's Disease
5

Loiasis
Q/Q(M)-473917 Report a Problem

Calabar swellings are a characteristic feature of which disease?
5

Loiasis
Calabar swellings are characteristic of Loiasis. They are transient, non-tender soft tissue swellings found
most commonly around joints.
Q/Q(M)-473917 Report a Problem

Erythema infectiosum is caused by a:
1

RNA virus
2

Herpes virus
3

Double stranded DNA virus
4

Single stranded DNA virus
5

None of these answers are correct
Q/Q(M)-473926 Report a Problem

Erythema infectiosum is caused by a:
4

Single stranded DNA virus
Erythema infectiosum is caused by parvovirus B19 which is a single stranded DNA virus.
Q/Q(M)-473926 Report a Problem

Which of the following is a gram positive rod?
1

Meningococcemia
2

Gonorrhea
3

Glanders
4

Melioidosis
5

Anthrax
Q/Q(M)-478742 Report a Problem

Which of the following is a gram positive rod?
5

Anthrax
Anthrax is a gram positive, spore-forming rod. Meningococcemia, gonorrhea, Glanders, and
Meloioidosis are all gram negative. Infection with anthrax initially begins after skin onoculation or may
follow after ingestion or inhalation of spores. Cutaneous anthrax initially begins as a localized infection
and subsequently evolves to toxemia. It appears as a painless papule that ay later become hemorrhagic
or nerotic. It produces two exotoxins: edema toxin and lethal toxin. Edema toxin is comprised of edema
11

factor and protective antigen. Lethal toxin is composed of lethal factor and protective antigen.
Q/Q(M)-478742 Report a Problem
Which of the following are potential forms of perinatally-acquired neonatal herpes disease?
1

Skin, eyes, mouth (SEM)
2

CNS
3

Disseminated
4

None of the above
5

All of the above
Q/Q(M)-482574 Report a Problem

Which of the following are potential forms of perinatally-acquired neonatal herpes disease?
5

All of the above
Perinatally acquired neonatal herpes disease can present in three different, occasionally overlapping,
forms: 1. skin, eyes, and mouth; 2. CNS; and 3. Disseminated, causing encephalitis, hepatitis,
pneumonia, and coagulopathy.
Q/Q(M)-482574 Report a Problem


A 16 year old female presents with a sore throat, painful cervical lymphadenopathy, and an enlarged
spleen. An empiric trial of amoxicillin leads to the development of a diffuse morbilliform rash. What is
the incubation period of the most likely viral cause of this patient\'s symptoms?
1

3-7 days
2

7-10 days
3

2-3 weeks
4

3-7 weeks
5

6 months
Q/Q(M)-482589 Report a Problem

A 16 year old female presents with a sore throat, painful cervical lymphadenopathy, and an enlarged
spleen. An empiric trial of amoxicillin leads to the development of a diffuse morbilliform rash. What is
the incubation period of the most likely viral cause of this patient\'s symptoms?
4

3-7 weeks
Infectious mononucleosis is characterized by a sore throat, painful lymphadenopathy, fatigue, and
splenomegaly. The formation of a morbilliform rash upon challenge with ampicillin or related
antibiotics is a classic finding. The most common virus associated with infectious mononucleosis is
Epstein-Barr virus. The incubation period is relatively long, 3-7 weeks.
Q/Q(M)-482589 Report a Problem

All of the following are potential causes of a false positive RPR except:
1

Systemic Lupus Erythematosus (SLE)
2

Pregnancy
3

Malignant Melanoma
12

4

Lepromatous Leprosy
5

Malaria
Q/Q(M)-473906 Report a Problem

All of the following are potential causes of a false positive RPR except:
3

Malignant Melanoma
Non-treponemal tests for syphilis measure antibodies against phospholipids released from treponemes
and damaged host cells. False positive non-treponemal tests can occur in the setting of pregnancy,
autoimmune diseases, other spirochete infections, and infectious diseases such as leprosy and malaria. It
does not occur in cases of melanoma.
Q/Q(M)-473906 Report a Problem
Hebra nose deformity is characteristic of infection with which organism?
1

Klebsiella pneumoniae
2

E. coli
3

Streptococcus pyogenes
4

Haemophilus influenzae
5

Serratia marcesens
Q/Q(M)-477608 Report a Problem

Hebra nose deformity is characteristic of infection with which organism?
1

Klebsiella pneumoniae
Hebra nose deformity is seen in Rhinoscleroma which is caused by Klebsiella pneumoniae.
Q/Q(M)-477608 Report a Problem

A middle-aged female presents with purplish pruritic plaques on her dorsal wrist and whitish plaques on
her buccal mucosa. A diagnosis of lichen planus is made. Which type of lichen planus is most strongly
associated with Hepatitis C virus infection?
1

Mucosal ulcerative lichen planus
2

Hypertrophic lichen planus
3

Lichen planus pemphigoides
4

Lichen planus of the nails
5

Vulvar lichen planus
Q/Q(M)-482598 Report a Problem

A middle-aged female presents with purplish pruritic plaques on her dorsal wrist and whitish plaques on
her buccal mucosa. A diagnosis of lichen planus is made. Which type of lichen planus is most strongly
associated with Hepatitis C virus infection?
1

Mucosal ulcerative lichen planus
Lichen planus presents a wide variety of cutaneous morphologies that share common histologic findings
of a lichenoid lymphocytic infiltrate, basal vacuolization, and dyskeratotic keratinocytes. The form of
lichen planus most strongly associated with Hepatitis C virus infection is mucosal ulcerative lichen
planus.
13

Q/Q(M)-482598 Report a Problem

A 21 year old female with a family history of C8 complement deficiency presents with sparsely
distributed hemorrhagic vesiculopustules on the palms, soles, and over joints. She has associated fever,
chills, arthralgias, and malaise. What is the treatment of choice for disseminated gonococcemia?
1

Ceftriaxone IV
2

Penicillin IM
3

Piperacillin-tazobactam IV
4

Levofloxacin PO
5

Azithromycin
Q/Q(M)-482693 Report a Problem
A 21 year old female with a family history of C8 complement deficiency presents with sparsely
distributed hemorrhagic vesiculopustules on the palms, soles, and over joints. She has associated fever,
chills, arthralgias, and malaise. What is the treatment of choice for disseminated gonococcemia?
1

Ceftriaxone IV
Gonococcemia is characterized by sparsely distributed hemorrhagic vesiculopustules on the palms,
soles, and over joints with associated fever, chills, arthralgias, and malaise. Recurrent cases associated
with complement deficiencies (C5-C8). Treatment is with IV ceftriaxone.
Q/Q(M)-482693 Report a Problem


A patient from India presents with fever, hepatosplenomegaly, and darkening of the skin. Which of the
following is a cause of visceral leishmaniasis?
1

L. donovani
2

L. major
3

L. mexicana
4

L. aethiopica
5

L. tropicana
Q/Q(M)-482698 Report a Problem

A patient from India presents with fever, hepatosplenomegaly, and darkening of the skin. Which of the
following is a cause of visceral leishmaniasis?
1

L. donovani
Visceral leishmaniasis is the most severe form of leishmaniasis. Complications can be fatal, and include
pneumonia, tuberculosis, and dysentery, which are potential opportunistic infections. Visceral
leishmaniasis is most commonly caused by L. donovani and L. infantum.
Q/Q(M)-482698 Report a Problem

Which of the extracellular enzymes of S. aureus plays a role in toxic shock syndrome (TSS)?
1

Coagulase
2

Penicillinase
3

Enterotoxin
14

4

Hemolysin
5

Exfoliatin
Q/Q(M)-480152 Report a Problem

Which of the extracellular enzymes of S. aureus plays a role in toxic shock syndrome (TSS)?
3

Enterotoxin
In addition to TSST-1 which is present in 50% of non-menstrual cases of TSS, enterotoxins, especially
B & C, cause TSS. These enterotoxins are also superantigens, which recognize the V-beta region of the
T-cell receptor. Coagulase clots plasma, Penicillinase degrades penicillin, Hemolysin lyses red blood
cells and exfoliatin splits the epidermis (antigen is desmoglein 1).
Q/Q(M)-480152 Report a Problem
A 19 year old sexually active male presents with a painful ulcer on the shaft of his penis. A biopsy
specimen stained with Giemsa reveals a "school of fish." Which of the following is the causative
organism?
1

Hemophilus ducreyi
2

Treponema pallidum
3

Calymmatobacterium granulomatis
4

Chlamydia trachomatis L1
5

Herpes simplex virus
Q/Q(M)-482695 Report a Problem

A 19 year old sexually active male presents with a painful ulcer on the shaft of his penis. A biopsy
specimen stained with Giemsa reveals a "school of fish." Which of the following is the causative
organism?
1

Hemophilus ducreyi
Chancroid is a sexually transmitted disease caused by the bacteria Hemophilus ducreyi. It is one of
many organisms that can cause a genital ulcer. Chancroid is characterized by a soft, painful chancre with
ragged edges. Giemsa stained sections reveal a "school of fish" configuration which is typical.
Treatment is with azithromycin, ceftriaxone, ciprofloxacin, or erythromycin.
Q/Q(M)-482695 Report a Problem

Neisseria gonorrheae would be identified with:
1

Sheeps blood agar
2

Agar supplemented with heme and nicotinamide
3

Chocolate agar with 10% CO2
4

Chocolate agar
5

Darkfield microscopy
Q/Q(M)-480160 Report a Problem


Neisseria gonorrheae would be identified with:
3

Chocolate agar with 10% CO2
15

Neisseria gonorrheae should be grown on chocolate agar in 10% CO2. ThayerMartin, Martin-Lewis
or New York City medium also allow for N. gonorrheae growth. Agar supplemented with heme (x-
factor) and nicotinamide (v-factor) is needed for Hemophilus influenzae cultures. Darkfield microscopy
is helpful in diagnosing infections with Treponema pallidum (syphilis). Sheeps blood agar is useful
for identifying hemolytic strains of streptococcus and staphylococcus.
Q/Q(M)-480160 Report a Problem

Corynebacterium minuttissimum is responsible for which of the following infections?
1

Majocchi's granuloma
2

Trichomycosis axillaris
3

Erysipelas
4

Malakoplakia
5

Erysipeloid
Q/Q(M)-477606 Report a Problem

Corynebacterium minuttissimum is responsible for which of the following infections?
2

Trichomycosis axillaris
Corynebacterium minutissimum is responsible for causing trichomycosis axillaris, a superficial bacterial
overgrowth of axillary hairs characterized by circumferential yellow (most common), red, or black
granular concretions. Majocchi's granuloma is a dermatophyte infection. Erysipelas is caused by beta-
hemolytic group A streptococcus. Malakoplakia is usually caused by S. aureus, P. aeruginosa, or E. coli.
Erysipeloid is caused by Erysipelothrix rhusiopathiae. Corynebacterium is also the causative organism
in erythrasma and pitted keratolysis.
Q/Q(M)-477606 Report a Problem
Which of the following tests is the most sensitive serologic test in primary syphilis?
1

FTA-ABS
2

VDRL
3

ELISA
4

RPR
5

MHA-TP
Q/Q(M)-482696 Report a Problem


Which of the following tests is the most sensitive serologic test in primary syphilis?
1

FTA-ABS
The fluorescent treponemal antibody absorption test is the most sensitive serologic test in primary
syphilis. The ELISA is the test of choice in early primary and congenital syphilis because it is the first to
become reactive. However, it is less sensitive in late primary syphilis because of reduction of IgM
production. The ELISA is 100% specific. MHA-TP is less sensitive than FTA-ABS during primary
syphilis.
Q/Q(M)-482696 Report a Problem

Which of the following are inconsistent with the diagnosis of staphylococcal scalded skin syndrome?
16

1

Epidermal changes are produced by exfoliative toxins of staphlococcus
2

Initial event is usually a localized staph infection
3

Swabs and cultures of fluid filled blisters overwhelmingly grow staph
4

Prognosis is good in children with low mortality when anitbiotics are administered early
5

Cell necrosis does not occur in staphylococcal scalded skin syndrome as it does in T.E.N
Q/Q(M)-478752 Report a Problem

Which of the following are inconsistent with the diagnosis of staphylococcal scalded skin syndrome?
3

Swabs and cultures of fluid filled blisters overwhelmingly grow staph
Staphylococcal scalded skin syndrome: Lesions extend far beyond areas of actual staphylococcal
infection, by action of the epidermolytic exotoxin elaborated by the staphylococcus in remote sites.
Usually the staphylococci are present at a distant focus such as the parynx, nose ear, or conjuctiva. If
cultures are taken they should be obtained fromthe mucous membranes because the skin erythema and
desquamation is due to the distant effects of the exfoliative toxin, unlike the sitaution in bullous
impetigo, where S. aureus is present in the lesions. Epidermal changes are produced by exfoliative
toxins of Staphylococcus. Inital event is usually a localized Staph infection. Prognosis is good in
children with low mortality when anitbiotics are administered earyl. Cell necrosis does not occur in SSS
as it does in toxic epidermal necrolysis.
Q/Q(M)-478752 Report a Problem


What are the three rapid grower mycobacteria and what is the current treatment of choice?
1

leprae, chelonae, abscessus; clarithromcyin
2

Tuberculum, abscessus, fortuitum; rifampin
3

Chelonae, asbcessus, fortuitum; clarithromycin
4

Lepra, fortuitum, abscessus; minocycline
5

Chelonae, absccessus, fortuitum; minocycline
Q/Q(M)-482853 Report a Problem

What are the three rapid grower mycobacteria and what is the current treatment of choice?
3

Chelonae, asbcessus, fortuitum; clarithromycin
These three are rapid grower mycobacteria and the treatment of choice is clarithryomycin. Minocycline
can also be used, but it considered second line in WHO treatment.
Q/Q(M)-482853 Report a Problem


The most common bacterial cause of purpura fulminans is:
1

S. aureus
2

P. aeruginosa
3

H. influenzae
4

E. coli
5

Group A streptococcus
17

Q/Q(M)-474535 Report a Problem

The most common bacterial cause of purpura fulminans is:
5

Group A streptococcus
Group A streptococcus is the leading bacterial cause of purpura fulminans.
Q/Q(M)-474535 Report a Problem

A patient develops herpes zoster in the V1 distribution. A few days after development of the classic
vesicular rash, her family brings her to the emergency room because of new onset seizures. She had
been complaining of confusion, headache, fatigue, and loss of appetite prior to the convulsions. What is
the diagnosis?
1

Syndrome of inappropriate anti-diuretic hormone
2

Ramsay-Hunt syndrome
3

Delayed contralateral hemiparesis
4

Post-zoster neuropathy
5

Intracerebral vasculitis
Q/Q(M)-482588 Report a Problem

A patient develops herpes zoster in the V1 distribution. A few days after development of the classic
vesicular rash, her family brings her to the emergency room because of new onset seizures. She had
been complaining of confusion, headache, fatigue, and loss of appetite prior to the convulsions. What is
the diagnosis?
1

Syndrome of inappropriate anti-diuretic hormone
The question stem describes signs and symptoms of hyponatremia. The syndrome of inappropriate
antidiuretic hormone secretion is a known, albeit rare, complication of varicella-zoster virus infection.
The other four answer choices are also known complications of varicella-zoster virus infection, but do
not cause the disease presentation as described.
Q/Q(M)-482588 Report a Problem

A 40-year-old male patient from South America with HIV (CD4+ <200) presents with new purple-red
papules and nodules on his face that have been progressively enlarging. A biopsy is performed, showing
lobular vascular proliferations with plump endothelial cells on H&E staining. Immunohistochemical
staining of skin tissue for human herpes virus 8 (HHV8) is negative. Upon closer inspection of H&E-
stained sections, faint purple granular interstitial deposits are noted in the dermis. Which of the
following tissue stains is most appropriate for further work-up?
1

Fite
2

Chloroacetate esterase
3

Warthin-Starry
4

Giemsa
5

Ziehl-Neelsen
Q/Q(M)-482822 Report a Problem

A 40-year-old male patient from South America with HIV (CD4+ <200) presents with new purple-red
papules and nodules on his face that have been progressively enlarging. A biopsy is performed, showing
lobular vascular proliferations with plump endothelial cells on H&E staining. Immunohistochemical
18

staining of skin tissue for human herpes virus 8 (HHV8) is negative. Upon closer inspection of H&E-
stained sections, faint purple granular interstitial deposits are noted in the dermis. Which of the
following tissue stains is most appropriate for further work-up?
3

Warthin-Starry
Causes of angiomatous papulonodules arising in immunocompromised patients include Kaposis
sarcoma, bacillary angiomatosis, and verruga peruana, as well as pyogenic granulomas and cherry
angiomas. Bartonella, a gram negative bacillus, appears as faint purple-blue granular dermal deposits on
routine H&E-stained sections. These deposits appear black upon Warthin-Starry staining. Prominent
endothelial cells may also be seen. Both B. henselae and B. quintana have been implicated in the
pathogenesis of bacillary angiomatosus, while B. bacilliformis is the causative agent in verruga peruana
(as well as Bartonellosis). Erythromycin is the first-line treatment for bacillary angiomatosus, while
fluoroquinolones are first-line agents for treatment of verruga peruana.
Q/Q(M)-482822 Report a Problem

A 1 year old female patient is brought in by her mother because of the development of warty growths
peri-anally. A clinical examination and biopsy confirms the diagnosis of peri-anal warts. In 12 month
old patients, what is the most common cause of acquisition of human papilloma virus.
1

Vertical transmission peri-natally
2

Sexual abuse
3

Contact with a wart-containing finger
4

Contact with a fomite
5

Aerosol transmission
Q/Q(M)-482612 Report a Problem


A 1 year old female patient is brought in by her mother because of the development of warty growths
peri-anally. A clinical examination and biopsy confirms the diagnosis of peri-anal warts. In 12 month
old patients, what is the most common cause of acquisition of human papilloma virus.
1

Vertical transmission peri-natally
In any child with peri-anal warts, the possibility of sexual contact must be excluded. Fortunately, in
infants up to 12 months of age, the most common route of acquisition of the human papilloma virus is
vertical transmission at the time of delivery.
Q/Q(M)-482612 Report a Problem


A 67 year old homeless man comes to clinic complaining of a painful lump along his lateral jawline
which has been present for many months. He has noticed that it occasionally drains a purulent material.
What is the most likely diagnosis?
1

Actinomyces israelii
2

Nocardia brasilensis
3

Actinomadura madurae
4

Streptomyces somaliensis
5

Mycobacteria tuberculosis
Q/Q(M)-482620 Report a Problem

19

A 67 year old homeless man comes to clinic complaining of a painful lump along his lateral jawline
which has been present for many months. He has noticed that it occasionally drains a purulent material.
What is the most likely diagnosis?
1

Actinomyces israelii
Actinomycosis is a chronic suppurative infection forming nodules and sinus tracts with an exudate
containing sulfur granules. Risk factors include poor oral hygiene, penetrating foreign bodies, and dental
procedures. The causative organism is Actinomyces israelii, an anaerobic gram-positive rod.
Q/Q(M)-482620 Report a Problem
All of the following are features of scarlet fever except:
1

Pharyngitis
2

Pastias lines
3

S. aureus infection
4

Circumoral pallor
5

Sandpaper-like texture
Q/Q(M)-477222 Report a Problem

All of the following are features of scarlet fever except:
3

S. aureus infection
Scarlet fever is primarily a disease of children with most cases occurring between the ages of 1 and 10
years of age. Streptococcus pyogenes is the causative organism. It produces the streptococcus pyrogenic
exotoxin which elicits the cutaneous manifestations by enhancing delayed type hypersensitivity to
streptococcal products. Clinical findings include fever, sore throat, headache, chills, sandpaper rash on
the trunks, strawberry tongue, and Pastias lines (linear petechial streaks found in flexural locations).
Q/Q(M)-477222 Report a Problem

Which human papillomavirus type is associated with giant condylomata of Bushke and Lowenstein
(Bushke-Lowenstein tumor)?
1

1
2

2
3

6
4

16
5

31
Q/Q(M)-474538 Report a Problem

Which human papillomavirus type is associated with giant condylomata of Bushke and Lowenstein
(Bushke-Lowenstein tumor)?
3

6
The Bushke-Lowenstein tumor is caused by HPV-6 and 11.
Q/Q(M)-474538 Report a Problem

The treatment of choice for Loiasis is:
1

Ivermectin
20

2

Diethylcarbamazine (DEC)
3

Permethrin
4

Malathion
5

Chloramphenicol
Q/Q(M)-473916 Report a Problem

The treatment of choice for Loiasis is:
2

Diethylcarbamazine (DEC)
The recommended treatment for loiasis is Diethylcarbamazine (DEC).
Q/Q(M)-473916 Report a Problem
The dominant organism present on oily regions of the skin is:
1

Gram positive coagulase negative staphylococcus
2

Staphylococcus aureus
3

Propionibacterium acnes
4

Corynebacterium spp.
5

Propionibacterium avidum
Q/Q(M)-480149 Report a Problem


The dominant organism present on oily regions of the skin is:
3

Propionibacterium acnes
The most common organism on sebaceous regions of human skin is Propionibacterium acnes. This is an
aerotolerant, anaerobic gram positive club-shaped diptheroid that lives in the depths of the sebaceous
follicle. It has a potent lipase which cleaves sebaceous triglycerides into glycerol and free fatty acids.
The free fatty acids lower the pH and are thought to retard the growth of pathogens such as S. aureus.
Other residents of oily regions are: Malassezia furfur, S. epidermidis, Micrococcus luteus and other
catalase positive aerobic cocci. S. aureus is a pathogen, and is not considered normal flora of any region
of the skin. The other options are found on dry or moist regions of the skin.
Q/Q(M)-480149 Report a Problem


Orf virus infection is also known as:
1

Ecthyma
2

Ecthyma contagiosum
3

Echtyma gangrenosum
4

Pseudocowpox
5

Milker's nodule
Q/Q(M)-478751 Report a Problem


Orf virus infection is also known as:
21

2

Ecthyma contagiosum
Orf virus infection is also known as Ecthyma contagiosum. Ecthyma Gangrenosum is caused by
Pseudomonas aeruginosa. Ecthyma is caused by Staphylococcal or streptococcal pyoderma.
Pseudocowpox and Milker's nodule are also known as paravaccinia and caused by udders of infected
cows.
Q/Q(M)-478751 Report a Problem


All of the following are true regarding tularemia except:
1

Caused by the gram negative Francisella tularensis
2

Ticks serve as a reservoir
3

No longer one of the CDC's list of reportable diseases
4

Presents as an acute febrile illness
5

Potential for use as a biologic weapon
Q/Q(M)-477375 Report a Problem


All of the following are true regarding tularemia except:
3

No longer one of the CDC's list of reportable diseases
Tularemia is a zoonotic disease caused by the gram-negative coccobacillus Francisella tularensis. It is
transmitted by arthropods (particularly tick bites) or from contact with infected mammals (usually
rabbits). It characteristically presents as an acute febrile illness with other variable clinical
manifestations depending on the route of inoculation. Possible symptoms could include an ulcer at the
site of inoculation, pharyngitis, ocular lesions, lymphadenopathy, and pneumonia. Diagnosis can be
made by culture or a fourfold titer change. Because of its potential use as a biologic agent, tularemia is
once again a reportable disease.
Q/Q(M)-477375 Report a Problem


The most common location of herpes gladiatorum is the:
1

Groin
2

Chest
3

Anterior thigh
4

Face
5

Hand
Q/Q(M)-473914 Report a Problem
The most common location of herpes gladiatorum is the:
4

Face
The face is the most common location for Herpes Gladiatorum.
Q/Q(M)-473914 Report a Problem


22

A 67-year-old man presents with grouped painful vesicles on an erythematous base on his lateral
sacrum. He denies any history of herpes simplex type I or II. He is angry with you for suggesting that he
may have herpes. What could explain this presentation?
1

He actually has shingles and your diagnosis is incorrect
2

A significant number of people are asymptomatic for HSV infection
3

He is lying
4

He has the early stages of Alzheimers disease
5

This presentation represents a primary infection with Epstein Barr Virus, (HHV4)
Q/Q(M)-480180 Report a Problem

A 67-year-old man presents with grouped painful vesicles on an erythematous base on his lateral
sacrum. He denies any history of herpes simplex type I or II. He is angry with you for suggesting that he
may have herpes. What could explain this presentation?
2

A significant number of people are asymptomatic for HSV infection
A significant number of people are asymptomatic for HSV infection but are seropositive for anti-HSV
antibodies. Lumbosacral HSV occurs in about 10% of HSV infections and is more common in older age
groups. In some cases, this may be the first presentation of HSV that the patient is aware of.
Shingles would be a reasonable consideration for the differential diagnosis and a direct
immunofluorescence scraping and viral culture would confirm that this typical presentation of HSV
actually is. It is possible that the patient is lying or that he has early Alzheimers disease, but less
likely. EBV does not present in this manner.
Q/Q(M)-480180 Report a Problem

A 52 year-old butcher presents with a tender lesion on his left hand as shown. The treatment of choice
for this condition is:
1

Penicillin
2

Azithromycin
3

Ciprofloxacin
4

Tetracycline
5

Chloramphenicol
Q/Q(M)-474913 Report a Problem

A 52 year-old butcher presents with a tender lesion on his left hand as shown. The treatment of choice
for this condition is:
23

1

Penicillin
The clinical description and lesion shown in the image suggest a diagnosis of erysipeloid, caused by
Erysipelothrix rhusiopathiae. The treatment of choice is penicillin.
Q/Q(M)-474913 Report a Problem


Which of the following vectors is responsible for transmitting Chagas disease?
1

Tstse fly
2

Reduviid bug
3

Black fly
4

Mosquito
5

Sandfly
Q/Q(M)-474541 Report a Problem

Which of the following vectors is responsible for transmitting Chagas disease?
2

Reduviid bug
The reduviid big is the vector of American trypanosomiasis (Chagas disease). The vector of African
trypanosomiasis is the tstse fly.
Q/Q(M)-474541 Report a Problem

Which of the following skin manifestations of M. tuberculosis infection is a tuberculid?
1

Erythema induratum of Bazin
2

Lupus vulgaris
3

Miliary TB
4

Scrofuloderma
5

Tuberculous gumma
Q/Q(M)-480176 Report a Problem

Which of the following skin manifestations of M. tuberculosis infection is a tuberculid?
1

Erythema induratum of Bazin
A tuberculid is a cutaneous immunologic reaction to tuberculosis elsewhere in the body. By definition,
cultures and stains for M. tuberculosis are negative. Erythema induratum of Bazin/Nodular vasculitis
present as erythematous or cyanotic nodules on the posterior calves. 85% of cases occur in women. The
other listed presentations of TB are all due to secondary spread from an established TB infection at
another location. In some cases, organisms can be recovered and cultures may be positive.
Q/Q(M)-480176 Report a Problem
A young male patient presents with a painful ulcer with a ragged edge after a new sexual encounter.
What is the treatment of choice?
1

Penicillin
2

Doxycycline
24

3

Azithromycin
4

Ciprofloxacin
5

Cephalexin
Q/Q(M)-482859 Report a Problem
A young male patient presents with a painful ulcer with a ragged edge after a new sexual encounter.
What is the treatment of choice?
3

Azithromycin
Ulcers in the genital area in sexually active patient generally fall into two groups: painful and painless.
Painless ulcers include syphilllis, lymphogranuloma venereum, and granuloma inguinale. The primary
cause of painful erosion or ulcer is H. ducreyi or herpes simplex. An ulcer with a ragged edge that is
painful is characteristic of H. ducreyi. The treatment of choice is azithromycin.
Q/Q(M)-482859 Report a Problem
The vector of Trench Fever is the:
1

Human body louse (Pediculus humanus corporis)
2

Cat flea (Ctenocephalides felis)
3

Sandfly (Phlebotamus perniciosus)
4

Rat flea (Xenopsylla cheopis)
5

Trombiculid mite
Q/Q(M)-473909 Report a Problem
The vector of Trench Fever is the:
1

Human body louse (Pediculus humanus corporis)
The human body louse (Pediculus humanus corporis)is the vector of Trench Fever, Epidemic Typhus,
and Relapsing Fever.
Q/Q(M)-473909 Report a Problem

Granuloma Inguinale is caused by:
1

Chlamydia trachomatis types I, II & III
2

Hemophilis ducreyi
3

Calymmatobacterium granulomatis
4

Treponema pertenue
5

Treponema pallidum
Q/Q(M)-480173 Report a Problem


Granuloma Inguinale is caused by:
3

Calymmatobacterium granulomatis
Granuloma inguinale is caused by an infection of Calymmatobacterium granulomatis. It is a
granulomatous skin disease of the genitals caused by sexual transmission. It is a gram negative rod that
is similar to the Enterobacteriaceae. It is a smll, raised papule or subcutaneous nodule that rapidly
ulcerates, rarely with lymphadenopathy. It is not self healing and spreads by radial extension.
25

Hemophilis ducreyi is the cause of chancroid, Chlamydia trachomatis causes Lymphogranuloma
Venereum, Treponema pertenue causes Yaws and T. pallidum is the cause of syphilis.
Q/Q(M)-480173 Report a Problem
Which Rickettsial infection has a negative Weil-Felix test?
1

Rocky Mountain Spotted Fever (R. rickettsii)
2

Mediterranean fever (R. conorii)
3

Epidemic typhus (R. prowazekii)
4

Endemic typhus (R. typhi)
5

Rickettsialpox (R. akari)
Q/Q(M)-476497 Report a Problem

Which Rickettsial infection has a negative Weil-Felix test?
5

Rickettsialpox (R. akari)
Rickettsia are short, gram-negative rods which are strict intracellular parasites. The bacteria is
transmitted via tick which much be attached for more than 6 hours. The Weil-Felix test exploits cross-
reactivity between the Proteus vulgaris antigen and RIckettsia.
Q/Q(M)-476497 Report a Problem


A young patient presents to you after participating in a bar brawl a few nights before with a painful
swollen erythematous right hand. The most likely organism is:
1

Staphylococus aureus
2

Eikenella corrodens
3

Pasturella multocida
4

Streptococcus pyogenes
5

Bacteriodes fragilis
Q/Q(M)-479611 Report a Problem
A young patient presents to you after participating in a bar brawl a few nights before with a painful
swollen erythematous right hand. The most likely organism is:
2

Eikenella corrodens
Eikenella corrodens is a member of normal oral flora, that is commonly the cause of human bite and fist
fight infections. Pasturella multocida is a common organism found in dog and cat bites. Staph and Strep
could be the right answers for a cellulitis, but with this history are not the correct choices. Baceriodes
fragilis is found in normal bowel flora.
Q/Q(M)-479611 Report a Problem



What characteristic color is seen on diascopy of this lesion?
1

Apple jelly
2

Blue
26

3

Orange
4

Red
5

Coral red
Q/Q(M)-476877 Report a Problem

What characteristic color is seen on diascopy of this lesion?
1

Apple jelly
Leishmaniasis recidivans is a type of Old World leishmaniasis. Clinically, it appears as a red papule
covered with white scale. On diascopy, it has a characteristic apple jelly color.
Q/Q(M)-476877 Report a Problem


Similar lesions were also seen on this infant's face. Treatment of choice is:
1

phototherapy
2

oral steroids
3

acyclovir
4

topical retinoic acid
5

oral erythromycin
Q/Q(M)-480500 Report a Problem

Similar lesions were also seen on this infant's face. Treatment of choice is:
27

3

acyclovir
This child has eczema herpeticum, a cutaneous infection with HSV which occurs in patients with pre-
existing skin disorders such as atopic dermatitis. It begins as clusters of umbilicated and hemorrhagic
vesicles in areas of abnormal skin. Lesions spread rapidly but can respond quickly to antiviral treatment.
Children with underlying AD who come into frequent contact with an individual with orolabial HSV
may have recurrent outbreaks of eczema herpeticum.
Q/Q(M)-480500 Report a Problem

What is the most common cause of erythema multiforme?
1

Herpes simplex virus
2

Mycoplasma pneumonia
3

Amoxicillin
4

Ibuprofen
5

Cytomegalovirus
Q/Q(M)-482809 Report a Problem

What is the most common cause of erythema multiforme?
1

Herpes simplex virus
The most common cause of erythema multiforme (EM) is herpes simplex virus, which may not be active
at the time of the EM eruption. Patients with recurrent EM are typically treated with acyclovir or
valacyclovir. Mycoplasma pneumonia is a cause of EM, but is not the most common. Amoxicillin,
ibuprofen, and cytomegalovirus may cause EM, but are not as common.
Q/Q(M)-482809 Report a Problem

The Jarisch-Herxeimer reaction is most closely associated with the release of which of the following
cytokines?
1

IFN-alpha
2

IL-12
3

TNF-alpha
4

IL-4
5

IL-10
Q/Q(M)-474528 Report a Problem

The Jarisch-Herxeimer reaction is most closely associated with the release of which of the following
cytokines?
3

TNF-alpha
TNF-alpha. The Jarisch-Herxheimer reaction is associated with increased plasma concentrations of
TNF-alpha after antibiotic treatment of syphilis. The Jarisch-Herxheimer reaction has also been
described following treatment of louse-borne relapsing fever, lyme disease and leptospirosis.
Q/Q(M)-474528 Report a Problem

A woman who handles fish tanks develops a slow-growing cluster of papules on the hand. What
temperature (Celsius) is most optimal for this organism's growth?
28

1

25 degrees
2

30 degrees
3

37 degrees
4

40 degrees
5

42 degrees
Q/Q(M)-482328 Report a Problem

A woman who handles fish tanks develops a slow-growing cluster of papules on the hand. What
temperature (Celsius) is most optimal for this organism's growth?
2

30 degrees
This patient most likely has contract Mycobacterium marinum, otherwise known as "fish tank
granuloma." On primary isolation M. marinum grows best at 30-33 degrees C in 7-21 days. Unlike
Mycobacterium tuberculosis, most strains of M. marinum will not grow at the usual incubation
temperature of 37 degrees C. Dimorphic fungi have both yeast and fungal forms: hyphae morphology at
25 degrees C and yeast phase at 37 degrees C.
Q/Q(M)-482328 Report a Problem

What is the principal vector of Lyme Disease in the Northeastern U.S.?
1

Ixodes ricinus
2

Soft-bodied ticks (Ornithodoros)
3

Ixodes dammini
4

Ambylomma americanum
5

Dermacentor variabilis
Q/Q(M)-473954 Report a Problem

What is the principal vector of Lyme Disease in the Northeastern U.S.?
3

Ixodes dammini
The proncipal vector of Lyme Disease in the Northeastern U.S. is Ixodes dammini (Ixodes scapularis).
Q/Q(M)-473954 Report a Problem
A 27 year-old woman who is 30 weeks pregnant presents with erythema migrans. The treatment of
choice for this patient is:
1

Doxycycline
2

Erythromycin
3

Chloramphenicol
4

Clindamycin
5

Amoxicillin
Q/Q(M)-473915 Report a Problem


A 27 year-old woman who is 30 weeks pregnant presents with erythema migrans. The treatment of
choice for this patient is:
29

5

Amoxicillin
In the setting of pregnancy, amoxicillin is the treatment of choice for Lyme Disease since doxycycline is
contraindicated.
Q/Q(M)-473915 Report a Problem

Unilateral palpebral and periorbital edema is characteristic of which of the following infectious
diseases?
1

African trypanosomiasis
2

American trypanosomiasis
3

Amebiasis
4

Schistosomiasis
5

Onchocerciasis
Q/Q(M)-482840 Report a Problem

Unilateral palpebral and periorbital edema is characteristic of which of the following infectious
diseases?
2

American trypanosomiasis
American trypanosomiasis (Chagas disease) is caused by infection with Trypanosoma cruzi via
Triatoma arthropod vectors (reduviid bugs) and is prevalent predominantly in Central and South
America. Transmission of this protozoal infection may occur following the bite of an infected reduviid
bug, who then defecates on human skin after feeding, allowing the metacyclic trypomastigotes entry into
the bite wound. Alternatively, infected feces may gain direct entry through mucosal surfaces, including
the conjunctivae. When the latter occurs, characteristic unilateral palpebral and periorbital edema
ensues, known as Romanas sign. Entry into the skin may result in a localized plaque of erythema
and induration with regional lymphadenopathy, termed a chagoma. Other systemic complications
resulting from chronic infection include cardiac arrhythmias, congestive heart failure, megacolon, and
megaesophagus. Treatment is with nifurtimox or benznidazole.
Q/Q(M)-482840 Report a Problem

A patient with HIV/AIDS develops disseminated flesh-colored papules with central umbilication. A
diagnosis of molluscum contagiosum is made. What is the CD4 cell count below which molluscum
contagiosum lesions are first seen in HIV patients?
1

100
2

200
3

400
4

800
5

1000
Q/Q(M)-482603 Report a Problem

A patient with HIV/AIDS develops disseminated flesh-colored papules with central umbilication. A
diagnosis of molluscum contagiosum is made. What is the CD4 cell count below which molluscum
contagiosum lesions are first seen in HIV patients?
1

100
HIV/AIDS patients are afflicted with a wide variety of opportunistic pathogens. The spectrum of
30

infectious diseases can be used to predict a patient\'s CD4 helper T cell count. Molluscum contagiosum
is usually not seen until a patient\'s CD4 count drops below 100.
Q/Q(M)-482603 Report a Problem

The Mikulicz is the histologic hallmark of which of the following conditions:
1

Leishmaniasis
2

Granuloma inguinale
3

Rhinoscleroma
4

Rhinosporidiosis
5

Histoplasmosis
Q/Q(M)-480168 Report a Problem

The Mikulicz is the histologic hallmark of which of the following conditions:
3

Rhinoscleroma
Rhinoscleroma is caused by Klebsiella rhinoscleromatis, a gram negative rod that causes a chronic
granulomatous infection of the upper respiratory tract. It is predominantly seen in Mexico, Central and
South America. The histologic hallmark is the Mikulicz cell, a large histiocyte that contains the bacteria
in its cytoplasm. Rhinosporidiosis is not an intracellular bacteria, but an extracellular fungi caused by
Rhinosporidium seeberi mainly seen in India, Sri Lanka and occasionally the southeastern United States.
Extremely large endospores are present in tissue. The other listed organisms are intracellular pathogens
but do not have the Mikulicz cell as a feature of infection.
Q/Q(M)-480168 Report a Problem
At what rate of speed does this parasite migrate through the skin?
1

0.1 cm/day
2

1 cm/day
3

2 cm/day
4

10 cm/day
5

100 cm/day
Q/Q(M)-476851 Report a Problem
31



At what rate of speed does this parasite migrate through the skin?
3

2 cm/day
The infection depicted is larva migrans or creeping eruption which is caused by Ancylostoma
braziliense. This nematode is typically a hookworm of cats and dogs. It migrates at a rate of 2 cm/day.
Larva currens migrates at a rate of 10 cm/day.
Q/Q(M)-476851 Report a Problem

All of the following are true regarding smallpox except:
1

Caused by variola virus
2

Transmitted by respiratory secretions
3

Virus is found in skin lesions
4

Vaccination is contraindicated in children under 5
5

Associated with thrombocytopenia
Q/Q(M)-477358 Report a Problem

All of the following are true regarding smallpox except:
4

Vaccination is contraindicated in children under 5
Smallpox is an acute exanthematous infection caused by the variolae poxvirus. It is transmitted
primarily by respiratory secretions as well as more infrequently skin inoculation and fomite spread.
Following contact, there is an asymptomatic period of 12-13 days. Following the asymptomatic period,
patients develop a prodrome of fever, headache, back pain and vomiting for 3 days. Then, the
characteristic deep seated papules that vesicles appear. These lesions mature into a pustule with a central
umbilication. Occasionally, patients may develop a hemorrhagic form of the disease which is associated
with thrombocytopenia and poor prognosis. Vaccination is not contraindicated in children under 5.
Q/Q(M)-477358 Report a Problem
32

You are in a medical mission in the Peruvian Amazonia and were asked to examine a patient. The
patient was concerned about the skin lesions on her abdomen. What is the most likely diagnosis in your
differential?
1

Yellow fever
2

Dengue fever
3

Malaria
4

Strongyloidosis
5

Leptospirosis
Q/Q(M)-482882 Report a Problem


You are in a medical mission in the Peruvian Amazonia and were asked to examine a patient. The
patient was concerned about the skin lesions on her abdomen. What is the most likely diagnosis in your
differential?
4

Strongyloidosis
Strongyloides stercoralis causes morphologically variable skin lesions. Rarely, a petechial purpuric
eruption which resembles multiple thumbprints may initially present over the periumbilical area. Later
on it may involve the whole abdomen, thighs, legs and arms. The petechial purpuric skin lesions in
disseminated strongyloidiasis are due to damage to the blood vessels caused by larval migration.
Q/Q(M)-482882 Report a Problem

A patient on the bone marrow transplant service has a fever, neutropenia, hemorrhagic bullae and
erythematous nodules with dusky gray centers. The organism most likely to be responsible is:
1

Streptococcus pyogenes
2

Mycobacterium tuberculosis
3

Candida albicans
4

Pseudomonas aeruginosa
5

Staphylococcus aureus
Q/Q(M)-477315 Report a Problem

A patient on the bone marrow transplant service has a fever, neutropenia, hemorrhagic bullae and
erythematous nodules with dusky gray centers. The organism most likely to be responsible is:
4

Pseudomonas aeruginosa
Ecthyma gangrenosum is an infection of critically ill or immunocompromised individuals by
Pseudomonas aeruginosa. Clinically, patients develop opalescent, tense vesicles or pustules surrounded
by a narrow pink to violaceous halo. The lesions rapidly become hemorrhagic, then violaceous and
necrotic leaving ulcers. The most common location is on the buttocks and lower extremities. Treatment
is with aminoglycosides.
Q/Q(M)-477315 Report a Problem
A 10-year-old boy develops an acute illness and rash along with marked enlargement of the posterior
cervical lymph nodes. This presentation is most consistent with:
1

West African sleeping sickness
33

2

East African sleeping sickness
3

Chagas disease
4

Schistosomiasis
5

Sparaganosis
Q/Q(M)-482313 Report a Problem

A 10-year-old boy develops an acute illness and rash along with marked enlargement of the posterior
cervical lymph nodes. This presentation is most consistent with:
2

East African sleeping sickness
East African sleeping sickness is caused by Trypanosoma rhodisiense and leads to acute illness with
rapid fatal course and pronounced posterior cervical lymphadenopathy (Winterbottom's sign). West
African sleeping sickness is more chronic. American trypanosomiasis, or Chagas disease, features
unilateral conjunctivitis and edema of the face (Romana's sign) and heart and gastrointestinal sequelae.
Schistomsomiasis causes swimmer's itch, while sparaganosis from ingestion of Spirometra leads to
painful edematous lumps.
Q/Q(M)-482313 Report a Problem

Which of the following is a pox virus?
1

Molluscum contagiosum
2

Herpes simplex virus
3

Pediculosis capitis
4

Sarcoptes scabei
5

Human immunodeficiency virus
Q/Q(M)-482808 Report a Problem


Which of the following is a pox virus?
1

Molluscum contagiosum
Of the answer choices, molluscum contagiosum is the only pox virus. Pediculosis capitis, the cause of
lice, and sarcoptes scabei, the cause of scabies are not viruses. Herpes simplex virus is a herpes virus
and HIV is a lentivirus.
Q/Q(M)-482808 Report a Problem

A patient recently had pharyngitis and now has an eruption of guttate psoriasis. Which of the following
tests would be helpful in determining that this patient had a Streptococcal infection?
1

A complete blood count
2

FTA-ABS
3

U1-RNP
4

DNAse B
5

Serum calcium
Q/Q(M)-480156 Report a Problem

34

A patient recently had pharyngitis and now has an eruption of guttate psoriasis. Which of the following
tests would be helpful in determining that this patient had a Streptococcal infection?
4

DNAse B
The DNAse B, hyaluronidase or streptolysin O (ASLO) antibodies in serum can be helpful for
confirming streptococcal infection in situations where streptococci cannot be isolated. The FTA-ABS is
used for detecting syphilis, and is the only test that is positive in the first 14 days of infection. U1-RNP
is an extractable nuclear antibody which is a marker for Mixed Connective Tissue Disease.
Q/Q(M)-480156 Report a Problem

A 23 year old intravenous drug user presents with the onset of purpura on the legs. Biopsy reveals
vasculopathic changes with intravascular thrombi seen in superficial dermal vessels. Serum studies for
cryoglobulins and Hepatitis C virus are both positive. Patients with HCV and mixed cryoglobulinemia
are more likely to develop what malignancy?
1

Non-Hodgkins lymphoma
2

Cutaneous T-cell lymphoma
3

Anaplastic large cell lymphoma
4

Kaposi\'s sarcoma
5

Acute myelocytic leukemia
Q/Q(M)-482597 Report a Problem

A 23 year old intravenous drug user presents with the onset of purpura on the legs. Biopsy reveals
vasculopathic changes with intravascular thrombi seen in superficial dermal vessels. Serum studies for
cryoglobulins and Hepatitis C virus are both positive. Patients with HCV and mixed cryoglobulinemia
are more likely to develop what malignancy?
1

Non-Hodgkins lymphoma
Mixed cryoglobulinemia is characterized by intravascular thrombi and the finding of monoclonal or
polyclonal IgG and IgM cryoglobulins in the blood. Mixed cryoglobulinemia is commonly associated
with HCV infection. Patients with HCV-associated mixed cryoglobulinemia are at an increased risk of
developing non-Hodgkins lymphoma, approximately 35 times the general population.
Q/Q(M)-482597 Report a Problem

The diagnosis of leishmaniasis can be confirmed by which of the following techniques?
1

Weil-Felix test
2

Warthin-Starry stain
3

Culture in Sabourad's agar
4

Culture in Novy-MacNeal-Nicolle (NNN) medium
5

Culture in Michel's medium
Q/Q(M)-473908 Report a Problem

The diagnosis of leishmaniasis can be confirmed by which of the following techniques?
4

Culture in Novy-MacNeal-Nicolle (NNN) medium
Confirmation of infection with Leishmaniasis is performed by culture in Novy-MacNeal-Nicolle (NNN)
medium.
Q/Q(M)-473908 Report a Problem
35


A child presents with fever, cutaneous tenderness and erythema of flexural and periorifacial areas.
Within 24 hours the erythema progresses to flaccid blisters and erosions. No organisms are cultured and
Nikolsky's sign is positive. First line therapy includes:
1

IV penicillinase resistant penicillin
2

Aspirin
3

IV IgG
4

PO tetracylcine
5

PO cephalexin
Q/Q(M)-481615 Report a Problem

A child presents with fever, cutaneous tenderness and erythema of flexural and periorifacial areas.
Within 24 hours the erythema progresses to flaccid blisters and erosions. No organisms are cultured and
Nikolsky's sign is positive. First line therapy includes:
1

IV penicillinase resistant penicillin
First line therapy for SSSS includes IV penicillinase resistant penicillin. Second line therapy is IV
macrolide treatment. Third line therapies are cephalosporin and vancomycin.
Q/Q(M)-481615 Report a Problem

An 18 year old male presents complaining of pruritus, erythema, and edema of the hands and feet. A
serum study for parvovirus antibodies is negative. What is the next most common cause of this
syndrome?
1

Epstein-Barr virus
2

Measles virus
3

Coxsackie virus
4

Cytomegalovirus
5

Human herpes virus 6
Q/Q(M)-482591 Report a Problem

An 18 year old male presents complaining of pruritus, erythema, and edema of the hands and feet. A
serum study for parvovirus antibodies is negative. What is the next most common cause of this
syndrome?
1

Epstein-Barr virus
Papular-purpuric gloves and socks syndrome is characterized by pruritus, erythema, and edema of the
hands and feet, primarily in teenagers and young adults. Parvovirus B19 is classically associated with
this eruption. While all of the viruses listed as answer choices have been associated with this syndrome;
Epstein-Barr virus is the best answer choice.
Q/Q(M)-482591 Report a Problem

A patient develops gastroenteritis after a trip to the seacoast. He reports eating raw oysters at his favorite
seafood restaurant. Which of the following organisms is the most likely cause of his gastrointestinal
problems?
1

Vibrio vulnificus
2

Vibrio parahemolyticus
36

3

Mycobacterium marinum
4

Erysipelothrix rhusiopathiae
5

Treponema carateum
Q/Q(M)-480161 Report a Problem

A patient develops gastroenteritis after a trip to the seacoast. He reports eating raw oysters at his favorite
seafood restaurant. Which of the following organisms is the most likely cause of his gastrointestinal
problems?
2

Vibrio parahemolyticus
Vibrio parahemolyticus is a cause of gastroenteritis after consuming contaminated seafood or shellfish.
V. vulnificus infection occurs in cuts/wounds that contact contaminated seawater or marine organisms.
M. marinum occurs after injury in an aquatic environment such as a fish tank or swimming pool.
Lesions typically occur on hands, elbows or knees and are characterized by painless inflammatory
nodules that may ulcerate and may occur in a sporotrichoid pattern. E. rhusiopathiae infection occurs
after injuries in fishermen, butchers or those handling raw meat. Initially there is burning pain at the
inoculation site followed by a gradually enlarging purple plaque with well defined margins. T. carateum
is the cause of Pinta.
Q/Q(M)-480161 Report a Problem

Which of the following forms of syphilis is characterized by ham-colored macules on the palms and
soles, condyloma lata, and \"moth-eaten\" alopecia?
1

Primary
2

Secondary
3

Latent
4

Tertiary
5

Congenital
Q/Q(M)-482694 Report a Problem

Which of the following forms of syphilis is characterized by ham-colored macules on the palms and
soles, condyloma lata, and \"moth-eaten\" alopecia?
2

Secondary
Secondary syphilis is characterized by ham-colored macules on the palms and soles, condyloma lata,
and \"moth-eaten\" alopecia, in addition to mucous patches and split papules. It typically lasts 4-12
weeks.
Q/Q(M)-482694 Report a Problem
Which of the following Rickettsial diseases would be rarely positive on Weil-Felix test?
1

Rocky Mountain Spotted Fever
2

Epidemic Typhus
3

Rickettsial pox
4

Endemic typhus
5

Scrub Typhus
Q/Q(M)-480179 Report a Problem
37



Which of the following Rickettsial diseases would be rarely positive on Weil-Felix test?
3

Rickettsial pox
Rickettsial pox is diagnosed by clinical suspicion and confirmatory biopsy, the Weil-Felix test is rarely
positive. The other listed Rickettsial diseases will have positive Weil-Felix tests. The Weil-Felix test
makes use of the cross-reaction between antigens from the gram-negative Proteus vulgaris species and
Rickettsia spp. It is not specific and rising titers in the appropriate clinical presentation is necessary for
confirmation. The Proteus antigens OX-K, OX-19 and OX-2 are employed.
Q/Q(M)-480179 Report a Problem

A 12 year old girl presents complaining of multiple warts on the fingers that have been recalcitrant to
common over-the-counter treatments. What is the clearance rate of untreated common warts in children
at 2 years after diagnosis?
1

25-30%
2

50-55%
3

70-75%
4

1%
5

5-10%
Q/Q(M)-482608 Report a Problem

A 12 year old girl presents complaining of multiple warts on the fingers that have been recalcitrant to
common over-the-counter treatments. What is the clearance rate of untreated common warts in children
at 2 years after diagnosis?
3

70-75%
While warts can be very difficult and frustrating to treat, the spontaneous clearance rate at two years is
actually quite high (70-75%).
Q/Q(M)-482608 Report a Problem

Roseola infantum is caused by which virus?
1

Coxsackie virus A16
2

Human herpes virus 6
3

Parvovirus B19
4

Epstein-Barr virus
5

Pox virus
Q/Q(M)-477135 Report a Problem

Roseola infantum is caused by which virus?
2

Human herpes virus 6
Roseola infantum may be cause be either human herpes virus types 6 or 7. It is the sixth of the
traditional exanthems of childhood. A diffuse, maculopapular eruption is typically preceded by a
prodrome of fever and either respiratory or gastrointestinal symptoms. The infection usually occurs in
the spring and the fall and nearly all children are seropositive for the virus by the age of 4.
Q/Q(M)-477135 Report a Problem
38


A 37-year-old man presents with a red, tender papule on his lateral right hand that subsequently
becomes necrotic with regional lymphadenopathy. An exanthem is also present. Which animal was this
individual hunting?
1

Rabbits
2

Grouse
3

Squirrels
4

Deer
5

Moose
Q/Q(M)-480165 Report a Problem

A 37-year-old man presents with a red, tender papule on his lateral right hand that subsequently
becomes necrotic with regional lymphadenopathy. An exanthem is also present. Which animal was this
individual hunting?
1

Rabbits
The presentation described above is that of Tularemia, caused by Francisella tularensis. The most
common form of Tularemia is the ulceroglandular form, which is described above. Infected animals,
most commonly rabbits, transmit the organism though occasionally tick bites or deer flies may be the
vector. Squirrels may be the vector of bubonic plague in endemic areas. The other animals would not
cause a syndrome as described.
Q/Q(M)-480165 Report a Problem
Hutchinsons teeth are a feature of which stage of syphilis?
1

Early congenital
2

Late congenital
3

Primary
4

Secondary
5

Tertiary
Q/Q(M)-474236 Report a Problem

Hutchinsons teeth are a feature of which stage of syphilis?
2

Late congenital
Hutchinsons teeth (widely-spaced, peg-shaped upper incisors) are a seen in Late Congenital Syphilis
(> 2 years of age).
Q/Q(M)-474236 Report a Problem

The mechanism of action of this virulence factor of Bacillus anthracis is via increasing the activity of
tumor necrosis factor alpha:
1

Lethal toxin
2

Edema toxin
3

Polyglutamate acid capsule
4

All of the above answers are correct
39

5

None of the above answers are correct
Q/Q(M)-477605 Report a Problem

The mechanism of action of this virulence factor of Bacillus anthracis is via increasing the activity of
tumor necrosis factor alpha:
1

Lethal toxin
Bacillus anthracis is responsible for causing Anthrax. Anthrax of the skin is characterized by a painless
edematous nodule which rapidly degenerates into an eschar. B. anthracis has 3 virulence factors: a
polyglutamate acid capsule which inhibits phagocytosis of the bacteria, edema toxin (edema factor and
protective factor) which results in edema secondary to induction of cAMP pathway, and lethal toxin
(lethal factor and protective factor) which leads to shock and death via increasing TNFalpha activity.
Q/Q(M)-477605 Report a Problem

Which of the following statements best describes the pathogenesis of scrofuloderma?
1

Hematogenous spread of M. tuberculosis from a distant site of infection
2

Autoinoculation of M. tuberculosis from advanced pulmonary tuberculosis
3

Contiguous spread from an underlying focus of tuberculous lymphadenitis
4

Primary incoculation of M. tuberculosis in a sensitized host
5

An cutaneous manifestation of military tuberculosis
Q/Q(M)-474530 Report a Problem


Which of the following statements best describes the pathogenesis of scrofuloderma?
3

Contiguous spread from an underlying focus of tuberculous lymphadenitis
Scrofuloderma is the result of contiguous spread onto skin from an underlying focus of tuberculous
infection. This typically occurs in a sensitized host with low immunity.
Q/Q(M)-474530 Report a Problem

Scrofuloderma is most commonly associated with an underlying infection of what organ system?
1

Lymph nodes
2

Spine
3

Lungs
4

Gastrointestinal tract
5

No underlying systemic disease
Q/Q(M)-482065 Report a Problem
40


Scrofuloderma is most commonly associated with an underlying infection of what organ system?
1

Lymph nodes
Scrofuloderma is a tuberculous or nontuberculous mycobacterial infection affecting children and young
adults, representing direct extension of tuberculosis into the skin from underlying structures such as
lymph nodes, bone, or lung. It most commonly presents as a neck abscess secondary to a tuberculosis
infection of the cervical lymph nodes. It is manifested by the development of painless subcutaneous
swellings that evolve into cold abscesses, multiple ulcers, and draining sinus tracts. The diagnosis is
based on positive Mantoux test/PPD test, histologic features of caseating granulomatous inflammation,
culture, and favorable response to anti-tuberculosis therapy.
Q/Q(M)-482065 Report a Problem

Which virus is most commonly associated with oral hairy leukoplakia?
1

Pox virus
2

Herpes virus
3

Epstein-Barr virus
4

Human papilloma virus
5

Parvovirus
Q/Q(M)-477142 Report a Problem

Which virus is most commonly associated with oral hairy leukoplakia?
3

Epstein-Barr virus
Oral hairy leukoplakia is an oral mucosal infection most often caused by the Epstein-Barr virus, which
occurs in immunocompromised patients. Clinically, thick, white plaques are noted on the lateral sides of
the tongue, often fissured.
Q/Q(M)-477142 Report a Problem

Measles is caused by a:
1

Paramyxovirus
2

Togavirus
3

Picornavirus
4

Rhabdovirus
5

Adenovirus
Q/Q(M)-473928 Report a Problem

Measles is caused by a:
41

1

Paramyxovirus
The measles virus is an RNA virus belonging to the Paramyxovirus family.
Q/Q(M)-473928 Report a Problem

A 45 year old female is admitted to the hospital because of symmetric, large ecchymotic areas with
irregular borders on the extremities, ears, and nose. Laboratory testing reveals the patient has
disseminated intravascular coagulation. Which of the following is the most common underlying
infection associated with this condition?
1

Group A streptococcus
2

Meningococcus
3

Staphylococcus
4

Pseudomonas
5

Mycobacterium
Q/Q(M)-482690 Report a Problem

A 45 year old female is admitted to the hospital because of symmetric, large ecchymotic areas with
irregular borders on the extremities, ears, and nose. Laboratory testing reveals the patient has
disseminated intravascular coagulation. Which of the following is the most common underlying
infection associated with this condition?
1

Group A streptococcus
Purpura fulminans is an oftentimes dramatic presentation of large ecchymotic areas of the skin, favoring
acral sites including the extremities, ears, and nose. The underlying pathophysiology is hemorrhagic
infarction of the skin due to disseminated intravascular coagulation. Group A streptococcal infection is
the leading underlying infection associated with this condition.
Q/Q(M)-482690 Report a Problem
What is the most common cause of focal epithelial hyperplasia?
1

Ebstein Bar virus(EBV)
2

Pachyonychia Congenita
3

human immunodeficiency virus(HIV)
4

human papilloma virus(HPV)
5

Dyskeratosis Congenita
Q/Q(M)-477635 Report a Problem

What is the most common cause of focal epithelial hyperplasia?
4

human papilloma virus(HPV)
Focal epithelia hyperplasia, also known as Heck's disease, is caused by HPV types 13, 32 and 57. It is
most commonly found on the lower lip but also on the buccal mucosa, gums and tongue. It is mainly a
disease of native Americans and Greenlander Eskimos. EBV causes an oral hairy leukoplakia in patients
with AIDS. Pachyonychia Congenita patients commonly develop a benign oral leukoplakia, while
Dykeratosis Congenita patients develop a premalignant oral leukoplakia.
Q/Q(M)-477635 Report a Problem

A patient presents with this anesthestic plaque. Which cytokines are upregulated in this disease?
42

1

IL-2
2

IL-4
3

IL-5
4

IL-10
5

All of these cytokines are upregulated
Q/Q(M)-476864 Report a Problem


A patient presents with this anesthestic plaque. Which cytokines are upregulated in this disease?
1

IL-2
Leprosy is caused by infections to Mycobacterium leprae. It is a acid fast bacilli. In tuberculoid leprosy,
Th1 cytokines (IL-2, IFN-gamma) are upregulated. Clinically, patients have few, well-circumscribed
anesthetic plaques.
Q/Q(M)-476864 Report a Problem

Which of the following is the vector for Dengue fever?
1

Phlebotomus papatasii
2

Culex mosquito
3

Aedes aegypti
4

Lutzomyia verrucarum
5

Xenopsylla cheopis
Q/Q(M)-478209 Report a Problem

Which of the following is the vector for Dengue fever?
3

Aedes aegypti
Dengue fever also known as "break-bone" fever is characterized by sudden high fever, backache, retro-
43

orbital pain, bone/joint pain, weakness, and malaise. It is caused by an arbovirus (RNA virus) which is
transmitted by Aedes aegypti, a species of mosquito.
Q/Q(M)-478209 Report a Problem

A 37 year old male with a history of HIV/AIDS presents complaining of the new onset of purplish
nodules of the feet and lower legs. He lost his insurance eight months ago and has been unable to afford
the HAART therapy that he was prescribed. HHV-8 is found in the semen of what percent of men with
this disease?
1

5%
2

20%
3

35%
4

50%
5

65%
Q/Q(M)-482592 Report a Problem

A 37 year old male with a history of HIV/AIDS presents complaining of the new onset of purplish
nodules of the feet and lower legs. He lost his insurance eight months ago and has been unable to afford
the HAART therapy that he was prescribed. HHV-8 is found in the semen of what percent of men with
this disease?
2

20%
Kaposi\'s sarcoma is a vascular neoplasm seen most commonly on the lower extremities, back, and
genitaliae. In the United States, this disease is most commonly seen in patients with AIDS. Kaposi\'s
sarcoma has been linked to infection with HHV-8. 20% of men with Kaposi\'s sarcoma will have
detectable HHV-8 in their semen.
Q/Q(M)-482592 Report a Problem

Where do you see Gamma-Favre bodies?
1

Granuloma inguinale
2

Lymphogranuloma venereum (LGV)
3

Syphilis
4

Chancroid
5

Herpes virus
Q/Q(M)-477444 Report a Problem

Where do you see Gamma-Favre bodies?
2

Lymphogranuloma venereum (LGV)
LGV is a sexually transmitted disease that is characterized by suppurative inguinal adenitis with matted
lymph nodes, inguinal bubo with secondary ulceration, and constitutional symptoms. It is caused by
Chlamydia trachomatis serotypes L1, L2, and L3. First line treatment is doxycycline 100mg bid for 3
weeks as well as treating the sexual partners. Gamma-Favre bodies are found in histiocytes in LGV.
Q/Q(M)-477444 Report a Problem

What is the following is the best choice for treatment of herpes zoster?
1

Ganciclovir
44

2

Foscarnet
3

Valacyclovir
4

Gabapentin
5

Cidofovir
Q/Q(M)-482817 Report a Problem

What is the following is the best choice for treatment of herpes zoster?
3

Valacyclovir
First line treatment of herpes zoster (shingles) is with valacyclovir, acyclovir, or famciclovir.
Gabapentin may be used for post-herpetic neuralgia. Cidofovir and ganciclovir are treatment options for
cytomegalovirus. Foscarnet is used to treat acyclovir resistant herpes simplex infections.
Q/Q(M)-482817 Report a Problem

A patient with primary syphilis is treated with Benzathine penicillin G. Soon after treatment, he
develops fever, headache, myalgias, and elevated white blood cell count, consistent with the Jarisch-
Herxheimer Reaction. This reaction is primarily mediated through which inflammatory cytokine?
1

TNF-alpha
2

IFN-gamma
3

IL-6
4

EGFR
5

IL-10
Q/Q(M)-482697 Report a Problem

A patient with primary syphilis is treated with Benzathine penicillin G. Soon after treatment, he
develops fever, headache, myalgias, and elevated white blood cell count, consistent with the Jarisch-
Herxheimer Reaction. This reaction is primarily mediated through which inflammatory cytokine?
1

TNF-alpha
The Jarisch-Herxheimer Reaction is characterized by fever, headache, lymphadenopathy, myalgias, and
elevated white blood cell count. It is caused by the release of inflammatory cytokines, particularly TNF-
alpha, due to phagocytosis of spirochetes following antibiotic administration.
Q/Q(M)-482697 Report a Problem

Acrodermatitis chronica atrophicans is caused by:
1

Borrelia burgdorferi
2

Borrelia burgdorferi senso stricto
3

Borrelia garinii
4

Borrelia afzelii
5

Unknown
Q/Q(M)-477428 Report a Problem

Acrodermatitis chronica atrophicans is caused by:
4

Borrelia afzelii
45

Acrodermatitis chronica atrophicans, also known as primary diffuse atrophy, is characterized by the
appearance on the extremities of diffuse reddish or bluish red, paper-thin skin allowing the blood vessels
to easily be seen beneath the skin. It is caused by Borrelia afzelii and is tick transmitted by Ixodes
ricinus. It is seen almost exclusively in Europe.
Q/Q(M)-477428 Report a Problem

Sun exposure has been associated with the development of what type of warts?
1

Flat warts
2

Myrmecial warts
3

Common warts
4

Butcher\'s warts
5

Condyloma accuminatum
Q/Q(M)-482609 Report a Problem

Sun exposure has been associated with the development of what type of warts?
1

Flat warts
Warts are common cutaneous lesions caused by the various human papilloma virus subtypes. Flat warts
are commonly caused by HPV subtypes 3, 10, 28, and 41. Sun exposure has been implicated as a
potential factor in the formation of flat warts.
Q/Q(M)-482609 Report a Problem

A 34 year old woman complains of recurrent painful blisters on her lower lip. These occur 4 times per
year, and are preceded by painful sensations for a day before the blisters appear. What is the most
common trigger of recurrent orolabial herpes simplex virus infection?
1

UVB radiation
2

UVA radiation
3

Visible light radiation
4

Emotional stress
5

Medication use, including opioids
Q/Q(M)-482619 Report a Problem
A 34 year old woman complains of recurrent painful blisters on her lower lip. These occur 4 times per
year, and are preceded by painful sensations for a day before the blisters appear. What is the most
common trigger of recurrent orolabial herpes simplex virus infection?
1

UVB radiation
Recurrent orolabial herpes simplex virus infection is a very common problem. In many cases, the cause
of the recurrences is idiopathic. Identifiable triggers include light exposure and emotional stress, among
others. UVB is the most commonly implicated factor.
Q/Q(M)-482619 Report a Problem

Woolsorters disease is caused by:
1

Erysipelothrix rhusiopathiae
2

Listeria monocytogenes
46

3

Corynebacterium minutissimum
4

Corynebacterium tenius
5

Bacillus anthracis
Q/Q(M)-480167 Report a Problem


Woolsorters disease is caused by:
5

Bacillus anthracis
Anthrax, also know as woolsorters disease or malignant pustule is caused by Bacillus anthracis, a
gram positive rod. It is primarily caused by contact with infected wild or domestic animals or their
products (such as wool). This organism has 3 clinical forms: inhalation, gastrointestinal and cutaneous.
The primary lesion is a malignant pustule which begins as a painless papule that evolves into a
hemorrhagic bulla. Ultimately, a black eschar forms. The lack of pain is helpful in distinguishing
between anthrax and other infections of the skin which are painful, such as ecthyma. The other listed
organisms are not causes of this disease.
Q/Q(M)-480167 Report a Problem

What named nerve is involved in the formation of Hutchinson's sign (vesicles at the nasal tip) in herpes
zoster?
1

Ophthalmic nerve
2

V2 (maxillary)
3

Nasociliary
4

Infratrochlear nerve
5

Labial nerve
Q/Q(M)-482583 Report a Problem

What named nerve is involved in the formation of Hutchinson's sign (vesicles at the nasal tip) in herpes
zoster?
3

Nasociliary
Hutchinson's sign is defined by vesicles at the nasal tip in a patient with facial herpes zoster. It is caused
by VZV invovlement of the nasociliary nerve. Patients suffering with this sign should be screened for
ophthalmologic herpes zoster invovlement.
Q/Q(M)-482583 Report a Problem

Herpes-associated erythema mutliforme is most commonly associated with which HLA type?
1

HLA B27
2

HLA DR2
3

HLA DR3
4

HLA B15
5

HLA B8
Q/Q(M)-477429 Report a Problem

Herpes-associated erythema mutliforme is most commonly associated with which HLA type?
47

4

HLA B15
Recurrent erythema multiforme (EM minor) is usually caused by recurrent herpes simplex, most
commonly HSV-1 orolabial disease. This is more correctly now called herpes-associated erythema
multiforme(HAEM) and has been more commonly associated with people with HLA type B15.
Q/Q(M)-477429 Report a Problem

The patient in this photograph has claw hands, hypoesthesia of the distal extremities, madarosis and
leonine facies. What type of leprosy does she have?
1

A. Lepromatous leprosy
2

Tuberculoid leprosy
3

Borderline leprosy
4

Erythema nodosum leprosum
5

Lucio phenomenon
Q/Q(M)-482071 Report a Problem

The patient in this photograph has claw hands, hypoesthesia of the distal extremities, madarosis and
leonine facies. What type of leprosy does she have?
1

A. Lepromatous leprosy
Hansen's disease (aka Leprosy) is a chronic granulomatous disease principally affecting the skin and
peripheral nervous system, caused by Mycobacterium leprae. The areas most commonly affected are the
superficial peripheral nerves, skin, mucous membranes of the upper respiratory tract, anterior chamber
of the eyes, and testes. These areas tend to be cool parts of the body. Tissue damage depends on the
degree to which cell-mediated immunity is expressed, the type and extent of bacillary spread and
multiplication, the appearance of tissue-damaging immunologic complications (ie, lepra reactions), and
the development of nerve damage and its sequelae.
Q/Q(M)-482071 Report a Problem

What is the causative agent of this parasitic eruption which also causes patchy pulmonary infiltrate and
eosinophilia?
1

Dracunculiasis
2

Gnathostomiasis
3

Larva currens
4

Larva migrans
5

Filariasis
Q/Q(M)-476852 Report a Problem
48


What is the causative agent of this parasitic eruption which also causes patchy pulmonary infiltrate and
eosinophilia?
4

Larva migrans
Larva migrans is caused by Ancylostoma braziliense, a hookworm of cats and dogs. It also associated
with Loeffler's syndrome, patchy infiltrate of the lungs with eosinophilia. Treatment for this parasitic
infection is ivermectin or albendazole.
Q/Q(M)-476852 Report a Problem

Foamy macrophages containing Klebsiella pneumoniae are called:
1

Virchow cells
2

Hansemann's cells
3

Mikulicz's cells
4

Michaelis-Gutman cells
5

Russell cells
Q/Q(M)-477425 Report a Problem

Foamy macrophages containing Klebsiella pneumoniae are called:
3

Mikulicz's cells
Mikuliczs cells are foamy macrophages found in Rhinoscleroma a chronic, inflammatory,
granulomatous disease of the upper respiratory tract. Virchow cells are found in Hansens disease.
Hansemann cells and Michaelis-Gutman bodies are found in Malakoplakia.
Q/Q(M)-477425 Report a Problem
Which of the following is not a tick born disease?
1

Lyme disease
2

Tularemia
3

Rocky Mountain spotted fever
4

Yaws
5

Ehrlichiosis
Q/Q(M)-478753 Report a Problem

49

Which of the following is not a tick born disease?
4

Yaws
Ticks are capable of transmitting:Lyme disease-Borrelia burgdorferi, a tickborne spirochete. Tularmia-
Francisella tularensis, bite of a tick. Rocky Mountain spotted fever-Rickettsia rickettsii, tick typhus.
Yaws-Treponema Palladium, subspecies pertunue-transmitted nonsexually, by contact with infectious
lesions. Ehrlichiosis-Ehrlichia chaffeensis, tickborne
Q/Q(M)-478753 Report a Problem

A 35 year old male has had frequent sex with prostitutes. He presents complaining of the rapid onset of a
new rash on his legs. A review of symptoms is otherwise unremarkable. Examination reveals palpable
purpura and tender nodules of the lower legs. Biopsy reveals vasculitis of small- and medium-sized
vessels. What percentage of patients with this disease are found to be infected with Hepatitis B virus?
1

7-8%
2

20%
3

35%
4

1%
5

50%
Q/Q(M)-482596 Report a Problem

A 35 year old male has had frequent sex with prostitutes. He presents complaining of the rapid onset of a
new rash on his legs. A review of symptoms is otherwise unremarkable. Examination reveals palpable
purpura and tender nodules of the lower legs. Biopsy reveals vasculitis of small- and medium-sized
vessels. What percentage of patients with this disease are found to be infected with Hepatitis B virus?
1

7-8%
Polyarteritis nodosa is characterized by the onset of painful nodules on the lower extremities. It can be
limited to the skin (cutaneous PAN), or can involve a wide variety of organ systems (systemic PAN).
Approximately 7-8% of cases of polyarteritis nodosa cases are associated with Hepatitis B virus
infection.
Q/Q(M)-482596 Report a Problem

Which of the following factors is most important in the skin providing protection against infectious
agents?
1

The mechanical barrier of intact skin
2

The resident flora present on intact skin
3

The relative dryness of intact skin
4

The cornified cell envelope
5

The presence of immunoglobulins on intact skin
Q/Q(M)-480150 Report a Problem
Which of the following factors is most important in the skin providing protection against infectious
agents?
1

The mechanical barrier of intact skin
The most important determinant of the skins protection against infectious agents is the mechanical
barrier of intact skin. The dryness of the skin is also important in retarding infection. The resident flora
50

may help prevent colonization by pathogenic organisms, but is not the most important factor.
Immunoglobulins are present on intact skin in miniscule amounts. The cornified cell envelope is not a
major determinant of infection prevention.
Q/Q(M)-480150 Report a Problem


Nonmenstrual cases of the staphylococcal toxic shock syndrome are mediated by which of the following
toxins?
1

Enterotoxins B and C
2

Exfoliative toxins A and B
3

Edema toxin
4

Epidermolytic toxins A and B
5

Pyrogenic exotoxins A, B, C
Q/Q(M)-474534 Report a Problem


Nonmenstrual cases of the staphylococcal toxic shock syndrome are mediated by which of the following
toxins?
1

Enterotoxins B and C
Toxic shock syndrome (TSS) is mediated by Enterotoxins B and C (50% of nonmenstrual cases) and
TSS toxin 1 (TSST-1), which is the leading cause of menstrual cases.
Q/Q(M)-474534 Report a Problem
What laboratory finding is commonly seen in children with measles virus infection?
1

Lymphopenia
2

Thrombocytopenia
3

Polycythemia
4

Anemia
5

Lymphocytosis
Q/Q(M)-482607 Report a Problem
What laboratory finding is commonly seen in children with measles virus infection?
1

Lymphopenia
Children who are actively infected with the measles virus are commonly found to have lymphopenia.
While an abnormality of any of the blood cell lines is possible during viral infections, in the case of
measles infections, lymphopenia is most common.
Q/Q(M)-482607 Report a Problem

A 48 year-old woman presents to the Emergency Room with a 2 day history of fever and a well
demarcated erythematous plaque on her right cheek. What is the treatment of choice?
1

Prednisone
2

Intravenous penicillin
3

Cephelexin
51

4

Fluconazole
5

Potent topical corticosteroid
Q/Q(M)-477603 Report a Problem

A 48 year-old woman presents to the Emergency Room with a 2 day history of fever and a well
demarcated erythematous plaque on her right cheek. What is the treatment of choice?
2

Intravenous penicillin
The most likely diagnosis is Erysipelas which is caused by beta-hemolytic group A streptococcus. The
treatment of choice is intravenous penicillin.
Q/Q(M)-477603 Report a Problem

The leading infectious cause of deafness and mental retardation in the U.S. is:
1

Congenital Rubella
2

Congenital varicella
3

Congenital Cytomegalovirus (CMV)
4

Congenital syphilis
5

Congenital herpes simplex
Q/Q(M)-473920 Report a Problem

The leading infectious cause of deafness and mental retardation in the U.S. is:
3

Congenital Cytomegalovirus (CMV)
Congenital CMV is the leading infectious cause of deafness and mental retardation in the U.S.
Q/Q(M)-473920 Report a Problem

Which of the following is a characteristic of papilloma viruses?
1

enveloped viruses
2

single stranded RNA viruses
3

replicate independent of the host cell
4

controlled by host antibody regulated response
5

can cause a condition that manifests with stridor and hoarseness in children
Q/Q(M)-482552 Report a Problem

Which of the following is a characteristic of papilloma viruses?
5

can cause a condition that manifests with stridor and hoarseness in children
Papilloma viruses are non-enveloped,double stranded DNA viruses. Cell mediated responses are
primarily responsible for controlling papillomavirus infections. Unlike viruses such as HSV, HPV does
not have enzymes required for replication of viral DNA, and is entirely dependent on the host cellular
machinery. Recurrent respiratory papillomatosis can be caused by HPV-6 and 11, with verrucous lesions
of the airways. It can occur as a juvenile or adult-onset form and present with hoarseness in children.
Unlike viruses such as HSV, HPV does not enzymes required for replication of viral DNA, and is
entirely dependent on the host cellular machinery for these functions.
Q/Q(M)-482552 Report a Problem
52

A patient with atopic dermatitis presents with frequent flares of dermatitis. They have multiple
lichenified plaques, some with serum crust. Which of the following organisms is most likely colonizing
these areas?
1

Aerobic diphtheroids
2

Gram positive coagulase negative cocci
3

Staphylococcus epidermidis
4

Staphylococcus aureus
5

Propionibacterium avidum
Q/Q(M)-480148 Report a Problem

A patient with atopic dermatitis presents with frequent flares of dermatitis. They have multiple
lichenified plaques, some with serum crust. Which of the following organisms is most likely colonizing
these areas?
4

Staphylococcus aureus
S. aureus rapidly becomes a member of the cutaneous flora in locations where there is serum or plasma
on the skin. Often patients with atopic dermatitis will have colonization of this organism in lesions and
may benefit from topical or oral antibiotics. P. avidum and aerobic diphtheroids are present in moist
regions of the skin. Gram positive coagulase negative cocci are the dominant organism in the dry
regions of the body. S. epidermidis is most commonly found in oily body regions.
Q/Q(M)-480148 Report a Problem


Which antibiotic is best to treat this condition caused by Bartonella hensalae.
1

Erythromycin
2

Ceftriaxone
3

Clindamycin
4

Trimethoprim/sulfamethoxasole
5

Fluconazole
Q/Q(M)-476525 Report a Problem


Which antibiotic is best to treat this condition caused by Bartonella hensalae.
1

Erythromycin
Bartonella hensalae is the causative agent of bacillary angiomatosis. The differential diagnosis of
bacillary angiomatosis may include pyogenic granulomas and Kaposi's sarcoma. The treatment of choice
is erythromycin.
Q/Q(M)-476525 Report a Problem
53



Which anatomic locations are most commonly involved in herpes gladiatorum?
1

Lateral neck, lateral face, forearm
2

Tips of fingers
3

Feet
4

Upper thighs and buttocks
5

Scalp
Q/Q(M)-482572 Report a Problem

Which anatomic locations are most commonly involved in herpes gladiatorum?
1

Lateral neck, lateral face, forearm
Herpes gladiatorum is most commonly seen on the lateral neck, lateral face, and forearm, areas which
most commonly contact a competing wrestler or the wrestling mat.
Q/Q(M)-482572 Report a Problem

What is the annual risk of developing herpes zoster in an HIV infected patient?
1

3% per month
2

10% per month
3

3% per year
4

10% per year
5

25% per year
Q/Q(M)-482580 Report a Problem
What is the annual risk of developing herpes zoster in an HIV infected patient?
3

3% per year
The annual risk of developing herpes zoster in an HIV infected individual, regardless of the CD-4 count,
is approximately 3%.
Q/Q(M)-482580 Report a Problem
The treatment of choice for acyclovir-resistant herpes simplex virus infection is:
1

Valcyclovir
2

Famciclovir
3

Indinivir
4

Saquinivir
5

Foscarnet
Q/Q(M)-474295 Report a Problem

The treatment of choice for acyclovir-resistant herpes simplex virus infection is:
5

Foscarnet
Foscarnet directly inhibits viral DNA polymerase (without requiring phosphorylation by TK) and is
therefore effective in acyclovir-resistant HSV infections. Cidofovir can also be used.
54

Q/Q(M)-474295 Report a Problem
A 48 year-old man develops headache, myalgias, and high fever 5 days after a hunting trip. On
examination he has a tender ulcer with raised margins and an eschar on his index finger, as well as
tender axillary lymphadenopathy. The most likely diagnosis is:
1

Anthrax
2

Orf
3

Glanders
4

Tularemia
5

Erysipeloid
Q/Q(M)-474536 Report a Problem

A 48 year-old man develops headache, myalgias, and high fever 5 days after a hunting trip. On
examination he has a tender ulcer with raised margins and an eschar on his index finger, as well as
tender axillary lymphadenopathy. The most likely diagnosis is:
4

Tularemia
Tularemia, which is caused by Francisella tularensis, occurs after exposure to infected animals,
including, rabbits, foxes, and squirrels. The clinical description is that of the ulceroglandular form,
which is the most common presentation.
Q/Q(M)-474536 Report a Problem

Milkers nodule is caused by infection with which of the following viruses?
1

Human herpesvirus-6 (HHV-6)
2

Human herpesvirus-8 (HHV-8)
3

Paravaccinia virus
4

Vaccinia virus
5

Orf virus
Q/Q(M)-474237 Report a Problem


Milkers nodule is caused by infection with which of the following viruses?
3

Paravaccinia virus
Milkers nodule is caused by paravaccinia virus, a poxvirus of the genus Parapoxvirus. It is transmitted
to humans from infected cows.
Q/Q(M)-474237 Report a Problem

Which of the following is the treatment of choice for the disease caused by Bartonella bacilliformis and
spread by the Lutzomyia sandfly?
1

Chloramphenicol
2

Erythromycin
3

Doxycyline
4

Minocycline
55

5

Azithromycin
Q/Q(M)-482685 Report a Problem

Which of the following is the treatment of choice for the disease caused by Bartonella bacilliformis and
spread by the Lutzomyia sandfly?
1

Chloramphenicol
Oroya Fever (Carrion\'s Disease) is caused by Bartonella bacilliformis. The Lutzomyia sandly is the
vector. While Bartonella infections respond to multiple classes of antibiotics, including tetracyclines and
macrolides, Oroya Fever has a high frequency of superinfection with Salmonella. Therefore,
chloramphenicol is the treatment of choice.
Q/Q(M)-482685 Report a Problem
A patient presents with fever, irritability and a scarlatiniform erythema with accentuation in the flexural
areas. Nikolsky positive blisters develop within 48 hours. Which of the following would predict the best
outcome?
1

Frequent NSAID use
2

Normal renal function
3

Normal immune system
4

Age < 5 years old
5

Age > 18 years old
Q/Q(M)-480153 Report a Problem

A patient presents with fever, irritability and a scarlatiniform erythema with accentuation in the flexural
areas. Nikolsky positive blisters develop within 48 hours. Which of the following would predict the best
outcome?
4

Age < 5 years old
Staphylococcal scalded skin syndrome is described in the question. This is predominantly a disease of
young children, < 5 years old. The mortality is low in children, but in adults it exceeds 50% due to
sepsis and electrolyte and fluid imbalance. NSAID use may predispose patients to a poorer outcome.
Normal renal function and immune system are helpful, but do not predict the best outcome.
Q/Q(M)-480153 Report a Problem


Tick-borne Relapsing Fever is caused by:
1

B. burgdorferi
2

B. recurrentis
3

B. duttonii
4

B. henselae
5

B. bacilliformis
Q/Q(M)-473913 Report a Problem
Tick-borne Relapsing Fever is caused by:
3

B. duttonii
56

Relapsing Fever can be either Louse-borne or Tick-borne. Tick-borne Relapsing Fever is caused by B.
duttonii, while Louse-borne Relapsing Fever is caused by B. recurrentis.
Q/Q(M)-473913 Report a Problem

Through which cellular protein does molluscum contagiosum evade host immunity?
1

IL-18 binding protein
2

Beta-4 integrin
3

T cell receptor
4

MHC II
5

Toll-like receptor 7
Q/Q(M)-482605 Report a Problem


Through which cellular protein does molluscum contagiosum evade host immunity?
1

IL-18 binding protein
Molluscum contagiosum virus, like other viral and bacterial pathogens, attempts to exploit the host
immune system to avoid destruction. In this case, molluscum contagiosum virus usesIL-18 binding
protein, which blocks the host helper T (Th1) response by reducing local IFN-gamma production.
Q/Q(M)-482605 Report a Problem



Exanthem Subitum is caused by which of the following?
1

Group A Steptococcus
2

Coxsackie virus
3

Human Herpes Virus-6 (HHV-6)
4

Parvovirus B19
5

Epstein-Barr Virus
Q/Q(M)-473919 Report a Problem

Exanthem Subitum is caused by which of the following?
3

Human Herpes Virus-6 (HHV-6)
Exanthem Subitum (Roseola Infantum, Sixth Disease) is caused by human herpesvirus 6, and 7 (HHV-6,
HHV-7).
Q/Q(M)-473919 Report a Problem
A 29-year-old woman presents with multiple crateriform pits on the soles of her feet following a month
of hiking on the Appalachian Trail. She wants a complete skin exam and wonders about her feet. Which
of the following do you plan to do next for her foot problem.
1

These pits are a variant of normal
2

You explain that she has a hereditary syndrome that causes these pits
3

You explain that this is a superficial bacterial infection and will resolve with topical therapy
57

4

You ask for a skin biopsy
5

You use a Woods lamp to confirm your diagnosis
Q/Q(M)-480169 Report a Problem

A 29-year-old woman presents with multiple crateriform pits on the soles of her feet following a month
of hiking on the Appalachian Trail. She wants a complete skin exam and wonders about her feet. Which
of the following do you plan to do next for her foot problem.
3

You explain that this is a superficial bacterial infection and will resolve with topical therapy
Multiple crateriform pits on the feet following suboptimal hygiene most likely is pitted keratolysis
caused by Micrococcus sedentarius. Treatment is topical erythromycin, clindamycin or benzoyl
peroxide. This is not Gorlins syndrome (Basal Cell Nevus syndrome). While these pits are common, it
is not a variant of normal. A skin biopsy could confirm the diagnosis, but is not necessary. A Woods
lamp would not be particularly helpful. Some believe that Corynebacterium spp. also cause this
condition, but that is still theory and not proven.
Q/Q(M)-480169 Report a Problem
A 12 month old boy has the sudden onset of a fever of 40 degrees Celsius. When the fever breaks
spontaneously in three days, an erythematous, morbilliform rash arises on the trunk and spreads to the
arms and legs. The rash is not itchy, and lasts 1-2 days. What is the most likely viral cause of this
syndrome?
1

HHV-6
2

Measles virus
3

Rubella virus
4

Coxsackie virus
5

Cytomegalovirus
Q/Q(M)-482595 Report a Problem


A 12 month old boy has the sudden onset of a fever of 40 degrees Celsius. When the fever breaks
spontaneously in three days, an erythematous, morbilliform rash arises on the trunk and spreads to the
arms and legs. The rash is not itchy, and lasts 1-2 days. What is the most likely viral cause of this
syndrome?
1

HHV-6
Roseola infantum is also known as exanthem subitum or sixth disease. It is seen in infants ages 6-36
months, and is characterized by the signs and symptoms as described. The syndrome is caused by
Human Herpes Virus 6.
Q/Q(M)-482595 Report a Problem
A 35 year-old woman who works in the animal product industry presents with a painless hemorrhagic
bulla. A diagnosis of anthrax is suspected. Gram stain of the vesicle fluid should reveal which of the
following?
1

Guarnieri bodies
2

Gram-negative cocci
3

Gram-negative bacilli
58

4

Gram-positive cocci
5

Gram-positive bacilli
Q/Q(M)-474914 Report a Problem

A 35 year-old woman who works in the animal product industry presents with a painless hemorrhagic
bulla. A diagnosis of anthrax is suspected. Gram stain of the vesicle fluid should reveal which of the
following?
5

Gram-positive bacilli
Anthrax is caused by Bacillus anthracis, which is a gram-positive spore-forming rod.
Q/Q(M)-474914 Report a Problem

A 35 year-old woman presents with a painful vaginal erosion and tender left-sided inguinal
lymphadenopathy. Gram stain reveals clusters of coccobacilli in a school of fish pattern. What is
the most likely diagnosis?
1

Primary syphilis
2

Lymphogranuloma venereum
3

Granuloma inguinale
4

Chancroid
5

Secondary syphilis
Q/Q(M)-474272 Report a Problem

A 35 year-old woman presents with a painful vaginal erosion and tender left-sided inguinal
lymphadenopathy. Gram stain reveals clusters of coccobacilli in a school of fish pattern. What is
the most likely diagnosis?
4

Chancroid
Chancroid classically presents with a soft, tender chancre with ragged edges and unilateral, tender
inguinal adenopathy (bubo). A school of fish pattern on Gram or Giemsa stain is diagnostic.
Q/Q(M)-474272 Report a Problem

A 28 year old sexually active female presents to clinic complaining of new growths of her labia majora.
Examination reveals condyloma acuminatum, and biopsy with immunohistochemistry confirms the
diagnosis of human papilloma virus infection. Condyloma accuminatum is most commonly caused by
what HPV type?
1

6 and 11
2

16 and 18
3

1 and 3
4

31 and 33
5

48 and 60
Q/Q(M)-482611 Report a Problem


A 28 year old sexually active female presents to clinic complaining of new growths of her labia majora.
Examination reveals condyloma acuminatum, and biopsy with immunohistochemistry confirms the
59

diagnosis of human papilloma virus infection. Condyloma accuminatum is most commonly caused by
what HPV type?
1

6 and 11
While many subtypes of HPV are associated with genital warts, 6 and 11 are the most common. In
general, potentially oncogenic HPV subtypes such as 16 and 18 cause sessile genital warts.
Q/Q(M)-482611 Report a Problem


What is the most common single nerve involved with herpes zoster?
1

Facial nerve
2

Trigeminal nerve
3

Spinal nerve C7
4

Spinal nerve T4
5

Spinal nerve T10
Q/Q(M)-482581 Report a Problem

What is the most common single nerve involved with herpes zoster?
2

Trigeminal nerve
While herpes zoster can be seen in the dermatome of any cranial or spinal nerve, the most common
single nerve involved is the trigeminal nerve.
Q/Q(M)-482581 Report a Problem

Epithelioma cuniculatum is an HPV-associated verrucous carcinoma involving the:
1

Oral mucosa
2

Scalp
3

Sole
4

Penis
5

Trunk
Q/Q(M)-473924 Report a Problem

Epithelioma cuniculatum is an HPV-associated verrucous carcinoma involving the:
3

Sole
Epithelioma cuniculatum is a form of verrucous carcinoma involving the sole.
Q/Q(M)-473924 Report a Problem

A 27 year-old HIV positive presents with this pruritic eruption? What is the characteristic cell type
found on histopathology?
1

Neutrophil
2

Plasma Cell
3

Basophils
4

Eosinophil
60

5

Lymphocyte
Q/Q(M)-476876 Report a Problem

A 27 year-old HIV positive presents with this pruritic eruption? What is the characteristic cell type
found on histopathology?
4

Eosinophil
Eosinophilic pustular folliculitis is a pruritic disorder that may be seen in HIV positive patients.
Typically, the CD4 count is < 300. In patients with HIV, pruritus tends to improve after institution of
anti-retroviral therapy.
Q/Q(M)-476876 Report a Problem

Xenopsylla cheopis transmits:
1

Epidemic typhus
2

Murine typhus
3

Rickettsia prowazekii
4

Scrub typhus
5

Rickettsia akari
Q/Q(M)-479608 Report a Problem

Xenopsylla cheopis transmits:
2

Murine typhus
Xenopsylla cheopis along with Ctenocephalides felis are fleas that transmit R. typhi, the organism
responsible for murine or endemic typhus. The body louse or Pediculus humanus corporis transmits
Rickettsia prowasekii the organism responsible for epidemic typhus. Scrub typhus is caused by R.
tsutsugamushi and is transmitted by chiggers or trombiculid mite larvae.
Q/Q(M)-479608 Report a Problem

A 23 year old Peace Corp volunteer returns from Africa with an extensive undermined, non-healing
ulceration on the right lower leg. Which of the following infections is the most likely cause of this
presentation?
1

M. marinum
2

M. kansasii
3

M. chelonei
4

M. ulcerans
5

Leishmania donovani
61

Q/Q(M)-480177 Report a Problem

A 23 year old Peace Corp volunteer returns from Africa with an extensive undermined, non-healing
ulceration on the right lower leg. Which of the following infections is the most likely cause of this
presentation?
4

M. ulcerans
The most likely cause of a non-healing ulceration in someone who has been in a tropical location is M.
ulcerans. M. marinum causes inflammatory nodules following injury in an aquatic environment. M.
chelonei is a rapidly growing organism most commonly found in soil, water, dust and animals which
presents as single/multiple erythematous SC nodules on an extremity or following a surgical procedure.
M. kansasii rarely causes skin lesions. L. donovani is a causse of visceral leishmaniasis in India and
Kenya (kala-azar).
Q/Q(M)-480177 Report a Problem

A patient is suspected to have contracted rabies after a dog bite. The best site for diagnostic biopsy
would be:
1

Neck
2

Acral area
3

Site of bite
4

Buccal mucosa
5

Buttocks
Q/Q(M)-482316 Report a Problem

A patient is suspected to have contracted rabies after a dog bite. The best site for diagnostic biopsy
would be:
1

Neck
The best site for diagnosis of rabies is a biopsy from the neck. This virus has an incubation period of 5
days to 1 year and demonstrates retrograde axoplasmic flow until it reaches the spinal cord. It
subsequently undergoes centrifugal spread along the peripheral nerves to the skin, intestine, and into the
salivary glands where it is shed.
Q/Q(M)-482316 Report a Problem

The treatment of choice for Oroya Fever is:
1

Penicillin
2

Doxycycline
3

Erythromycin
4

Minocycline
5

Chloramphenicol
Q/Q(M)-473910 Report a Problem

The treatment of choice for Oroya Fever is:
5

Chloramphenicol
The treatment of choice for Oroya Fever is chloramphenicol because of frequent superinfection with
Salmonella.
62

Q/Q(M)-473910 Report a Problem

An otherwise healthy six-day-old boy is brought into clinic by his mother with the complaint of tiny
thin-roofed vesicles with a rim of inflammation and a few lesions covered with a honey-colored
crust. Which organism is the most likely cause in this newborn?
1

Listeria monocytogenes
2

Group B streptococcus
3

Group D streptococcus
4

Staphylococcus Aureus
5

Lancefield type O organisms
Q/Q(M)-480158 Report a Problem

An otherwise healthy six-day-old boy is brought into clinic by his mother with the complaint of tiny
thin-roofed vesicles with a rim of inflammation and a few lesions covered with a honey-colored
crust. Which organism is the most likely cause in this newborn?
2

Group B streptococcus
Group B streptococcus is S. agalactae, commonly isolated from normal vaginal mucosa. In neonates,
impetigo caused by group B streptococci may develop. Listeria monocytogenes is a small, gram-positive
organism that may present as petechial, papular or pustular eruptions in acutely ill infants, usually those
that were meconium stained at birth. The usual cause of ecthyma is Group A streptococcus, though other
organisms can occasionally cause similar cutaneous infections. Group D streptococcus/Lancefield type
O organisms are enterococci. S. aureus can secondarily infect a lesion of ecthyma but is not the usual
cause of this condition.
Q/Q(M)-480158 Report a Problem

A 25-year old man with a history of a healed genital ulcer develops a diffuse papulosquamous eruption
resembling pityriasis rosea. What is the most likely time lapse between the appearance of the genital
lesion and the appearance of this eruption?
1

3-12 weeks
2

1-2 weeks
3

3-5 months
4

6-12 months
5

1-2 years
Q/Q(M)-482229 Report a Problem

A 25-year old man with a history of a healed genital ulcer develops a diffuse papulosquamous eruption
resembling pityriasis rosea. What is the most likely time lapse between the appearance of the genital
lesion and the appearance of this eruption?
1

3-12 weeks
The lesions of secondary syphilis are often diffusely distributed due to spirochetemia. Papulosquamous
lesions are characteristic, and the presence of constitutional symptoms such as fever, fatigue, headaches,
and bone pain may help distinguish secondary syphilis from pityriasis rosea. Other lesions of secondary
syphilis include moth-eaten alopecia, mucous patches, split papules, and condyloma lata. The
mucocutaneous manifestations of secondary syphilis typically occur 3-12 weeks after the appearance of
the chancre and last 4-12 weeks.
63

Q/Q(M)-482229 Report a Problem

Herpangina is caused by which of the following?
1

Group A coxsackievirus
2

Epstein Barr Virus
3

Parvovirus B19
4

Human Herpesvirus 6 (HHV-6)
5

Human Herpesvirus 6 (HHV-7)
Q/Q(M)-473921 Report a Problem

Herpangina is caused by which of the following?
1

Group A coxsackievirus
Herpangina is caused by Group A coxsackievirus.
Q/Q(M)-473921 Report a Problem

A 69 year old man who had recently returned from a boating trip in the Atlantic developed pain with
hemorrhagic bullae of his right leg. He had substained a cut on his right leg prior to leaving for the trip,
but swam in the water anyway. Which of the following treatments should be started?
1

Doxycycline
2

Erythromycin
3

Penicillin
4

Rifampin
5

Dapsone
Q/Q(M)-482551 Report a Problem

A 69 year old man who had recently returned from a boating trip in the Atlantic developed pain with
hemorrhagic bullae of his right leg. He had substained a cut on his right leg prior to leaving for the trip,
but swam in the water anyway. Which of the following treatments should be started?
1

Doxycycline
The case presentation describes an infection with Vibrio vulnificus, a Gram-negative rod, which may
lead to a rapidly expanding cellulitis, with hemorrhagic bullae and necrosis, and life-threatening
septicemia. This infection mainly occurs along the Atlantic seacoast. It may be acquired after ingesting
raw oysters or other seafood. Alternatively, localized skin infection may result after exposure of an open
wound to seawater. Doxycycline together with ceftazidime is the treatment of choice. In patients with
preexisting hepatic dysfunction or immunocompromise and whose wounds are exposed to or acquired in
saltwater, prophylactic antibiotic coverage with doxycycline, 100 mg every 12 h, and cleansing with
0.025% sodium hypochlorite solution may prevent progressive infection.
Q/Q(M)-482551 Report a Problem

What is the first-line recommended therapy for orf?
1

Penicillin
2

Erythromycin
3

Doxycycline
64

4

Bactrim
5

No therapy is needed since the condition is self-resolving
Q/Q(M)-482200 Report a Problem

What is the first-line recommended therapy for orf?
5

No therapy is needed since the condition is self-resolving
No therapy is needed, since orf is usually a benign, self-resolving infection. Orf is caused by a
parapoxvirus, often associated with sheep and goats. The infection undergoes 6 different stages of
evolution: the papular, target, acute, regenerative, papillomatous, and regressive stages.
Q/Q(M)-482200 Report a Problem

Blueberry Muffin Baby can be a feature of all of the following EXCEPT:
1

TORCH
2

Klippel-Trenaunay-Weber syndrome
3

Hemolytic disease of the newborn
4

Rhabdomyosarcoma
5

Langerhans cell histiocytosis
Q/Q(M)-473933 Report a Problem

Blueberry Muffin Baby can be a feature of all of the following EXCEPT:
2

Klippel-Trenaunay-Weber syndrome
Blueberry muffin lesions can be seen in the setting of prenatal infections (e.g. TORCH), severe anemia
(e.g. Hemolytic Disease of the newborn) and neoplastic diseases (e.g. rhabdomyosarcoma). It is not
associated with the Klippel-Trenaunay-Weber syndrome.
Q/Q(M)-473933 Report a Problem

A 36 year-old homeless man presents with a tender suppurative nodule on the mandible. "Sulfur
granules" are present on microscopy. The most likely diagnosis is:
1

Actinomycetoma
2

Actinomycosis
3

Anthrax
4

Acne Conglobata
5

Aspergillosis
Q/Q(M)-481612 Report a Problem

A 36 year-old homeless man presents with a tender suppurative nodule on the mandible. "Sulfur
granules" are present on microscopy. The most likely diagnosis is:
2

Actinomycosis
The presentation of a suppurative nodule discharging sulfur granules on the jawline of a patient with
poor oral hygeine is characteristic of actinomycosis - caused by the anaerobic gram-positive rod,
Actinomyces israelii. This differs from Actinomycetoma, which is a slowly progressive subcutaneous
infection characterized by tumefaction, draining sinuses, and an exudate containing grains on
microscopy. Unlike Eumycetomas - caused by fungi - Actinomycetomas are bacterial infections. The
65

foot is the most common location, followed by the thoracic area.
Q/Q(M)-481612 Report a Problem

What is the most common subtype of molluscum contagiosum virus infection seen in HIV patients?
1

MCV-1
2

MCV-2
3

MCV-3
4

MCV-4
5

MCV-5
Q/Q(M)-482602 Report a Problem

What is the most common subtype of molluscum contagiosum virus infection seen in HIV patients?
2

MCV-2
The most common subtype of molluscum contagiosum virus is MCV-1. In HIV patients, however,
MCV-2 is more common.
Q/Q(M)-482602 Report a Problem

Most common causative agent for the lesion shown in this 5-year old child is
1

Staphylococcus aureus
2

group A -hemolytic streptococci
3

Pseuomonas
4

kleibsiella
5

H.influenza
Q/Q(M)-482119 Report a Problem

Most common causative agent for the lesion shown in this 5-year old child is
1

Staphylococcus aureus
The picture shown is Impetigo, which is a common, contagious superficial skin infection caused most
commonly by staphylococci. Although seen in all age groups, the disease is most common in infants and
children. Lesions may involve any body surface but occur most frequently on the exposed parts of the
body, especially the face, hands, neck, and extremities. There are two classic forms of impetigo, bullous
and nonbullous. Nonbullous impetigo accounts for more than 70% of cases. Historically was caused
primarily by group A -hemolytic streptococci (GABHS), but now appears to be most commonly
caused by S. aureus. Anaerobic organisms may also be recovered from lesions of nonbullous impetigo.
(Reference: Brook I, Frazier EH, Yeager JK. Microbiology of nonbullous impetigo. Pediatr Dermatol
1997;14(3):192-195.)
66

Q/Q(M)-482119 Report a Problem

A patient had a severe febrile illness with hemolytic anemia and now has exophytic nodules on the
trunk, extremities and face. Which of the following organisms is the cause?
1

Bartonella henselae
2

Bartonella quintana
3

Bartonella bacilliformis
4

Rickettsia rickettsii
5

Rickettsia akari
Q/Q(M)-480163 Report a Problem

A patient had a severe febrile illness with hemolytic anemia and now has exophytic nodules on the
trunk, extremities and face. Which of the following organisms is the cause?
3

Bartonella bacilliformis
The first stage of this condition is known as Carrions disease, which can be fatal. It was in Dr.
Carrion, who self inoculated himself to prove that the cause of verruga peruana (the second stage
described above) was the same as that of the first stage. The cause of both Carrions disease and
verruga peruana is Bartonella bacilliformis. B. henselae is the cause of Cat Scratch disease while B.
quintana causes Trench Fever. Rickettsia rickettsii is the cause of Rocky Mountain Spotted Fever and
Rickettsia akari causes Rickettsial pox.
Q/Q(M)-480163 Report a Problem
An 18-year old man presents to the Dermatology Clinic with a nontender penile erosion that has been
present for 2 weeks. An indurated border and nontender bilateral inguinal lymphadenopathy are also
noted. What is the most likely diagnosis?
1

Primary syphilis
2

Chancroid
3

Herpes simplex
4

Lymphogranuloma venereum
5

HIV
Q/Q(M)-474235 Report a Problem
An 18-year old man presents to the Dermatology Clinic with a nontender penile erosion that has been
present for 2 weeks. An indurated border and nontender bilateral inguinal lymphadenopathy are also
noted. What is the most likely diagnosis?
1

Primary syphilis
A painless chancre with an indurated border is characteristic of primary syphilis. Associated painless
lymphadenopathy ("buboes") is also a common feature.
Q/Q(M)-474235 Report a Problem
A patient with AIDS develops umbilicated papules consistent with molluscum contagiosum. What
anatomic sites are favored for molluscum papules in patients with HIV?
1

Face and genitalia
2

Head and neck
3

Abdomen and back
67

4

Upper and lower extremities
5

Chest and shoulders
Q/Q(M)-482604 Report a Problem
A patient with AIDS develops umbilicated papules consistent with molluscum contagiosum. What
anatomic sites are favored for molluscum papules in patients with HIV?
1

Face and genitalia
Molluscum contagiosum is a common opportunistic infection of HIV patients. Lesions can be seen
anywhere, but the most common locations are on the face and genitalia.
Q/Q(M)-482604 Report a Problem
A patient with a chronic nose-bleed is treated with nasal packing. The packing is accidentally left in
place, and he develops Toxic Shock Syndrome. What is the implicated toxin?
1

Enterotoxin B
2

Toxic Shock Syndrome toxin 1
3

Exfoliative Toxin A
4

Edema toxin
5

Lethal factor
Q/Q(M)-482689 Report a Problem
A patient with a chronic nose-bleed is treated with nasal packing. The packing is accidentally left in
place, and he develops Toxic Shock Syndrome. What is the implicated toxin?
1

Enterotoxin B
Toxic Shock Syndrome is characterized by fever, hypotension, and a generalized scarlatiniform eruption
followed by desquamation. Menstrual-associated TSS is generally driven by Staphylococcus aureus,
which released Toxic Shock Syndrome toxin 1. In nonmenstrual cases, however, Enterotoxins B and C
are more commonly implicated, although TSST-1 has also been associated. Exfoliative toxins A and B
are involved in bullous impetigo, and edema toxin and lethal factor are involved in Bacillus anthracis
infections.
Q/Q(M)-482689 Report a Problem
Congenital Varicella Syndrome occurs after maternal varicella infection during which stage of
pregnancy?
1

First 20 weeks
2

Third trimester
3

5 days before and 2 days after delivery
4

20-24 weeks
5

None of these answers are correct
Q/Q(M)-473936 Report a Problem
Congenital Varicella Syndrome occurs after maternal varicella infection during which stage of
pregnancy?
1

First 20 weeks
Congenital Varicella Syndrome occurs after maternal varicella-zoster virus infection early in pregnancy
(up to 20 weeks gestation).
Q/Q(M)-473936 Report a Problem
Hecks disease is associated with which of the following types of human papillomavirus?
68

1

1
2

3
3

7
4

6/11
5

13/32
Q/Q(M)-480162 Report a Problem
Hecks disease is associated with which of the following types of human papillomavirus?
5

13/32
Hecks disease (focal epithelial hyperplasia) is caused by HPV 13 and 32 infections. These lesions are
multiple circumscribed papules on the oral mucosa that may resemble oral condyloma. They are not pre-
cancerous. HPV-1 is associated with plantar wars, while HPV-3 is associated with flat warts. HPV-7
cause butchers warts and are believed to be acquired from handling meat, poultry and fish.
Q/Q(M)-480162 Report a Problem
Which subtype of the human papilloma virus is most likely to have caused this infection?
1

HPV, type 1
2

HPV, type 2
3

HPV, type 3
4

HPV, type 5
5

HPV, type 6
Q/Q(M)-476853 Report a Problem
Which subtype of the human papilloma virus is most likely to have caused this infection?
5

HPV, type 6
The human papilloma virus is a member of the Papovavirus
family, a double-stranded DNA virus. HPV, type 6 has been implicated in giant conduloma of Buschke
and Lowenstein and anogenital condyloma.
Q/Q(M)-476853 Report a Problem
A rabbit farmer presents with fevers, myalgia, and a rash. A serum study reveals antibodies to
Francisella tularensis. Which of the following is the most common form of tularemia?
1

Ulceroglandular
2

Chancriform
3

Oculoglandular
4

Typhoidal
5

Meningeal
69

Q/Q(M)-482691 Report a Problem
A rabbit farmer presents with fevers, myalgia, and a rash. A serum study reveals antibodies to
Francisella tularensis. Which of the following is the most common form of tularemia?
1

Ulceroglandular
Tularemia represents a wide range of clinical syndromes caused by Francisella tularensis. The bacterium
is most commonly acquired through contact with infected animals, including rabbits. There are a wide
variety of variants, with the ulceroglandular form being most common. Ulceroglandular tularemia is
characterized by a chancre-like ulcer with raised borders and regional lymphadenopathy, most
commonly on the finger or hand. Treatment is with streptomycin.
Q/Q(M)-482691 Report a Problem
Coxsackievirus A16 is implicated in the pathogenesis of which of the following dieases?
1

Exanthem subitum
2

Fifth disease
3

Papular purpuric gloves and socks syndrome
4

Erythema infectiousum
5

Hand-Foot-and-Mouth disease
Q/Q(M)-474545 Report a Problem
Coxsackievirus A16 is implicated in the pathogenesis of which of the following dieases?
5

Hand-Foot-and-Mouth disease
Hand-Foot-and-Mouth disease is caused by coxsackievirus A16. Group A coxsackievirus infection is
also associated with herpangina.
Q/Q(M)-474545 Report a Problem
Streptobacillus moniliformis is the causative organism of which infectious disease?
1

Rat-bite fever (Haverhill fever)
2

Scrub typhus
3

Tularemia
4

Glanders
5

Cat scratch disease
Q/Q(M)-474262 Report a Problem
Streptobacillus moniliformis is the causative organism of which infectious disease?
1

Rat-bite fever (Haverhill fever)
Rat-bite fever (Haverhill fever) is caused by Streptobacillus moniliformis.
Q/Q(M)-474262 Report a Problem
A patient is given a single dose of dimethylcarbamazine and soon develop edema, itching, fever,
arthralgias, and exacerbation of pruritus. He most likely has which underlying condition?
1

Onchocerciasis
2

Loiasis
3

Filariasis
4

Gnothostomiasis
5

Trichinosis
Q/Q(M)-482312 Report a Problem
70

A patient is given a single dose of dimethylcarbamazine and soon develop edema, itching, fever,
arthralgias, and exacerbation of pruritus. He most likely has which underlying condition?
1

Onchocerciasis
This patient developed an acute reaction to dimethylcarbamazine indicating a Mazzotti reaction. This
reaction is seen with onchocerciasis. Appropriate treatment for onchocerciasis, caused by Onchocerca
volvulus, is ivermectin. These patients may demonstrate pruritus, dermatitis, leopard skin,
onchocercomas (nodules with microfilariae) and blindness.
Q/Q(M)-482312 Report a Problem
Which of the following is not a DNA virus?
1

Herpesvirus
2

Poxvirus
3

Parvovirus
4

Paramyxovirus
5

Adenovirus
Q/Q(M)-482571 Report a Problem
Which of the following is not a DNA virus?
4

Paramyxovirus
Paramyxovirus is a single-stranded RNA virus. All other listed choices are DNA-based viruses.
Q/Q(M)-482571 Report a Problem
Dracunculiasis is typically acquired from:
1

Soil
2

Salt water
3

Drinking water
4

Chiggers
5

Black flies
Q/Q(M)-474540 Report a Problem
Dracunculiasis is typically acquired from:
3

Drinking water
Dracunculiasis, which is caused by Dracunculus medinensis, is acquired by ingestion of copepod-
infested water.
Q/Q(M)-474540 Report a Problem
The causative organism in Whitmore disease is which of the following?
1

Streptobacillus moniliformis
2

Vibrio vulnificus
3

Klebsiella pneumoniae
4

Pseudomonas aeruginosa
5

Burkholderia pseudomallei
Q/Q(M)-477611 Report a Problem
The causative organism in Whitmore disease is which of the following?
5

Burkholderia pseudomallei
71

Whitmore disease also known as Meloidosis is characterized by pulmonary disease, septicemia, and
miliary abscesses. It is caused by Burkholderia pseudomallei.
Q/Q(M)-477611 Report a Problem
Carrion's disease is characterized by fevers, headaches and arthralgias and is accompanied by severe
hemolytic anemia. Superinfection with which organism is the most frequent cause of death?
1

Bartonella bacilliformis
2

Bartonella henselae
3

Bartonella Quintana
4

Salmonella
5

Shigella
Q/Q(M)-478757 Report a Problem
Carrion's disease is characterized by fevers, headaches and arthralgias and is accompanied by severe
hemolytic anemia. Superinfection with which organism is the most frequent cause of death?
4

Salmonella
Carrion's disease (Oroya fever) is characterized by fevers, headaches and arthralgias and is accompanied
by severe hemolytic anemia. Superinfection with Salmonella is the most frequent cause of death.
Protection from sandfly bites is all-important.
Q/Q(M)-478757 Report a Problem
Which of the following is the causative organism of cat-scratch disease?
1

B. quintana
2

B. henselae
3

B. bacilliformis
4

B. duttonii
5

B. burgdorferi
Q/Q(M)-474544 Report a Problem
Which of the following is the causative organism of cat-scratch disease?
2

B. henselae
B. henselae is the cause of cat-scratch disease. Humans are infected by a cat bite or scratch, while the cat
flea is responsible for cat to cat transmission.
Q/Q(M)-474544 Report a Problem
Most common causative agent for the lesion shown in image for this 34-year old immunocompetent man
is
1

Staphaylococcus aureus
2

Group A Beta hemolytic streptococcus
3

Pseudomonas aeruginosa
4

Klebsiella
72

5

Proteus
Q/Q(M)-482118 Report a Problem
Most common causative agent for the lesion shown in image for this 34-year old immunocompetent man
is
2

Group A Beta hemolytic streptococcus
Ecthyma is a deep or ulcerative type of pyoderma commonly seen on the lower extremities and buttocks
and caused most often by GABHS, in addition to staphyloccous aureus which also can be involved in
the infection. Ecthyma can present as small punched-out ulcers or a deep spreading ulcerative process.
The disorder begins in the same manner as impetigo, often following infected insect bites or minor
trauma, but penetrates through the epidermis to produce a shallow ulcer. The initial lesion is a
vesiculopustule with an erythematous base and firmly adherent crust. Removal of the crust reveals a
lesion deeper than that seen in impetigo, with an underlying saucer-shaped ulcer and raised margin. The
lesions are painful and heal slowly over a few weeks, often with scar formation. Ecthyma gangrenosum
is a cutaneous finding that may be seen in patients with Pseudomonas aeruginosa bacteremia. Most of
the affected individuals have an underlying immunodeficiency (either congenital or acquired) or a
history of cancer chemotherapy. Neutropenia may be a risk factor for ecthyma gangrenosum
Q/Q(M)-482118 Report a Problem
Botryomycosis is:
1

A fungal infection caused by M. canis
2

A bacterial infection commonly associated with Clostridium botulinum
3

A bacterial infection commonly associated with Staphylococcus aureus
4

A fungal infection caused by T. tonsurans
5

An acute, disseminated infection involving the genitourinary system
Q/Q(M)-478759 Report a Problem
Botryomycosis is:
3

A bacterial infection commonly associated with Staphylococcus aureus
Botryomycosis is an uncommon, chronic, idonlent disorder characterized by nodular, crusted, purulent
lesions. Sinuses that discharge sulfur granules are present. These heal with atrophic scars. The granules
yield most commonly Staphylococcus aureus on culture, although cases caused by Pseudomonas
aeruginosa, E. coli, Proteus, Bacteroides, and Streptococcus have been reported
Q/Q(M)-478759 Report a Problem
Which of the following organisms is spread by non-sexual person-to-person contact?
1

Treponema carateum
2

Chlamydia trachomatis types I, II & III
3

Hemophilis ducreyi
4

Calymmatobacterium granulomatis
73

5

Treponema pallidum
Q/Q(M)-480174 Report a Problem
Which of the following organisms is spread by non-sexual person-to-person contact?
1

Treponema carateum
Treponema carateum is the cause of Pinta which is primarily transmitted by direct contact. Occasional
insect vectors have also been implicated, but sexual contact is not the primary cause of spread. The other
options (C. trachomatis, H. ducreyi, C. granulomatis and T. pallidum) are all venereal diseases.
Q/Q(M)-480174 Report a Problem
Which of the following manifestations of syphilis would you expect to be present 2-6 months after the
individual was exposed?
1

Osteitis
2

Aortitis
3

Tabes dorsalis
4

Condylomata lata
5

Pseudochancre redux
Q/Q(M)-480172 Report a Problem


Which of the following manifestations of syphilis would you expect to be present 2-6 months after the
individual was exposed?
4

Condylomata lata
All of the listed options except condylomata lata are manifestations of tertiary syphilis, which does not
manifest for years after the initial infection. Secondary syphilis presents 2-6 months following infection.
Condylomata lata are moist, flat and smooth. Do not confuse them with Condyloma accuminatum which
are usually dry, cauliflower-like projections on the genitalia.
Q/Q(M)-480172 Report a Problem


A patient with known chronic Hepatitis C virus infection presents with hyperkeratotic plaques of the
bilateral feet. Biopsy reveals psoriasiform changes with superficial epidermal necrosis. Diet
supplementation with what mineral has been shown to improve the appearance and symptoms of this
cutaneous finding?
1

Zinc
2

Biotin
3

Magnesium
4

Calcium
5

Chromium
Q/Q(M)-482599 Report a Problem


A patient with known chronic Hepatitis C virus infection presents with hyperkeratotic plaques of the
bilateral feet. Biopsy reveals psoriasiform changes with superficial epidermal necrosis. Diet
supplementation with what mineral has been shown to improve the appearance and symptoms of this
cutaneous finding?
74

1

Zinc
Necrolytic acral erythema is characterized by hyperkeratotic well-defined plaques on the lower
extremities, seen occasionally in patients with hepatitis C virus infection. Treatment consists primarily
of treating the underlying infection. Diet supplementation with zinc has been shown to improve
necrolytic acral erythema, even in the presence of normal serum zinc levels.
Q/Q(M)-482599 Report a Problem

A 43 year-old man presents with suppurative nodules and sinus tracts over the mandible. He also is
noted to have poor oral hygiene. What is the most likely causative organism?
1

Bartonella Henselae
2

Nocardia brasilensis
3

Streptococcus somaliensis
4

Actinomyces israelii
5

Staphylococcus aureus
Q/Q(M)-478207 Report a Problem


A 43 year-old man presents with suppurative nodules and sinus tracts over the mandible. He also is
noted to have poor oral hygiene. What is the most likely causative organism?
4

Actinomyces israelii
Actinomyces israelii is an anaerobic gram-positive rod which causes chronic suppurative nodules and
sinus tracts with an exudate containing sulfur granules. It most commonly effects the cervicofacial area,
especially near the mandible. The abdomen and thoracic areas can also be affected. The source of
infection is endogenous and patients with poor oral hygiene, penetrating foreign bodies, and recent
dental procedures are at increased risk. Treatment of choice is penicillin.
Q/Q(M)-478207 Report a Problem

The most common extracutaneous complications of varicella zoster virus is:
1

Lymphoreticular
2

Musculoskeletal
3

Cardiovascular
4

Central nervous system
5

Genitourinary
Q/Q(M)-481898 Report a Problem



The most common extracutaneous complications of varicella zoster virus is:
4

Central nervous system
Zoster usually resolves without sequelae in children and young adults with intact immune systems.
However, the pain, cutaneous eruption and complications of zoster become more severe with increasing
age and immune compromise. Complications of zoster include post-herpetic neuralgia (PHN), secondary
75

bacterial infection, scarring, ophthalmic zoster, Ramsay-Hunt syndrome, meningoencephalitis, motor
paralysis, pneumonitis and hepatitis. Reference: Stalkup JR et al. Human Herpesviruses. In: Bolognia
JL, Jorizzo JL, Rapini RP, eds. Dermatology. New York: Mosby, 2003:1243.
Q/Q(M)-481898 Report a Problem


Which exotoxin(s) are involved in mediating the effects of Bacillus anthracis - the causative organism
for anthrax?
1

Exotoxin B
2

Edema Toxin
3

Alpha toxin
4

Lethal Toxin
5

Edema Toxin and Lethal Toxin
Q/Q(M)-473951 Report a Problem


Which exotoxin(s) are involved in mediating the effects of Bacillus anthracis - the causative organism
for anthrax?
5

Edema Toxin and Lethal Toxin
B. anthracis has 2 exotoxins: edema toxin and lethal toxin, each comprised of a pair of noncovalently
linked proteins: Edema toxin = Edema Factor (EF) + Protective Antigen (PA) Lethal toxin =
Lethal Factor (LF) + PA EF causes gelatinous edema of anthrax skin lesions by inducing an increase
in cyclic adenosine monophosphate (cAMP) levels LF causes shock and death in disseminated
anthrax via the release of tumor necrosis factor-alpha (TNF-a)and interleukin-1b (IL-1b) PA is
responsible for entry of exotoxins into the cell by receptor-mediated endocytosis.
Q/Q(M)-473951 Report a Problem
Name the syndrome defined by gonorrhea with perihepatitis and liver adhesions.
1

Lhermitte-Duclos Syndrome
2

Turcot Syndrome
3

Fitz-Hugh-Curtis Syndrome
4

Leser-Trelat Syndrome
5

Clutton's Syndrome
Q/Q(M)-482484 Report a Problem


Name the syndrome defined by gonorrhea with perihepatitis and liver adhesions.
3

Fitz-Hugh-Curtis Syndrome
Gonorrhea, caused by a gram negative intracellular diplococcus, often presents cutaneously with
hemorrhagic vesiculopustules of the distal extremities. Septic arthritis is also an important presentation
of gonococcemia, most commonly involving the knee.
Q/Q(M)-482484 Report a Problem


76

Which of the following is true regarding lymphogranuloma venereum?
1

The primary stage presents as a painful herpetiform ulcer.
2

Proctocolitis can be seen in the late stages of this disease.
3

It is caused by Chlamydia psittaci.
4

Buboes should be incised and drained.
5

Transmission does not occur via skin-to-skin contact.
Q/Q(M)-482504 Report a Problem


Which of the following is true regarding lymphogranuloma venereum?
2

Proctocolitis can be seen in the late stages of this disease.
Lymphogranuloma venereum presents as a painless ulcer and is caused by Chlamydia trachomatis
serotypes L1, L2, L3. While buboes can be aspirated, they should not be incised and drained.
Transmission occurs through sexual contact, but can occur via skin-to-skin contact. Complications of
late stage disease include proctocolitis, perirectal abscesses, rectovaginal fistulas, and anal fistulas.
Q/Q(M)-482504 Report a Problem

Pediculus humanus var. corporis (human body louse) is the vector in which of the following diseases:
1

Endemic typhus (R. typhus)
2

Epidemic typhus (R. prowazekii)
3

Q fever (C. burnetii)
4

Rickettsialpox (R. akari)
5

Rocky Mountain Spotted Fever (R. rickettsii)
Q/Q(M)-478208 Report a Problem


Pediculus humanus var. corporis (human body louse) is the vector in which of the following diseases:
2

Epidemic typhus (R. prowazekii)
The human body louse is the implicated Trench fever, epidemic typhus, and relapsing fever. Trench
fever usually affects alcoholic men and is manifested by fevers. Treatment is with ceftriaxone,
erythromycin, or doxycycline. Epidemic typhus is manifested by fevers, chills, malaise, and a pink
macular eruption beginning in the axilla and trunk. Treatment is with tetracycline or chloramphenicol.
Relapsing fever is manifested by paroxysmal fevers, headache, lymphocytoma, and
erythematous/petechial macules on trunk/extremities. Treatment is with doxycycline.
Q/Q(M)-478208 Report a Problem



Treatment of severe nodulocystic acne with which of the following vitamin derivatives may completely
arrest the disease process through decreasing P. acnes?
1

Vitamin D
2

Vitamin A
77

3

Vitamin K
4

Vitamin E
5

Vitamin B3
Q/Q(M)-480159 Report a Problem
Treatment of severe nodulocystic acne with which of the following vitamin derivatives may completely
arrest the disease process through decreasing P. acnes?
2

Vitamin A
13-cis-retinoic acid (isotretinoin/Accutane) is a derivative of vitamin A. It has several mechanisms of
action in acne. It rapidly suppresses sebum production, causing a decrease in P. acnes populations. It
also decreases follicular plugging by an uncertain mechanism. Vitamins D, K and E are fat soluble
vitamins, but are not able to completely arrest the disease process in acne. Vitamin B3 is niacin and is
associated with a diet of exclusively of corn, millet or sorghum. Other sources: carcinoid, Hartnup
disease, parasites, GI disorders, IV nutrition, and psychiatric disease. Findings include the 3 Ds:
Diarrhea, Dementia, Dermatitis.
Q/Q(M)-480159 Report a Problem

Which of the following diseases is caused by a virus?
1

Rocky Mountain spotted fever
2

Human monicytic ehrlichiosis
3

Colorado tick fever
4

Boutonneuse spotted fever
5

Q fever
Q/Q(M)-478900 Report a Problem

Which of the following diseases is caused by a virus?
3

Colorado tick fever
Rocky Mountain spotted fever is caused by Rickettsia rickettsii. Human monicytic ehrlichiosis is caused
by Ehrlichia chaffeensis. Colorado tick fever is caused by Reoviiridae. Boutonneuse fever is caused by
Rickettsia conorii. Q fever is caused by Coxiella burnetti.
Q/Q(M)-478900 Report a Problem


The usual culture medium for mycobacteria is:
1

Chocolate agar in 10% CO2
2

Lowenstein-Jensen
3

New York City
4

Sheeps blood agar
5

Agar supplemented with heme and nicotinamide
Q/Q(M)-480175 Report a Problem
78


The usual culture medium for mycobacteria is:
2

Lowenstein-Jensen
The Lowenstein-Jensen medium is used to culture mycobacteria most commonly. Neisseria gonorrheae
is be grown on chocolate agar in 10% CO2. Thayer Martin, Martin-Lewis or New York City medium
also allow for N. gonorrheae growth. Agar supplemented with heme (x-factor) and nicotinamide (v-
factor) is needed for Hemophilus influenzae cultures. Sheeps blood agar is useful for identifying
hemolytic strains of streptococcus and staphylococcus.
Q/Q(M)-480175 Report a Problem
A male patient experiences painful vesicles on the palate, tongue, and ipsilateral eardrum. Additionally,
he complains of vertigo and difficulty with speaking and eating. Involvement of what anatomic structure
is responsible for this constellation of symptoms?
1

Geniculate ganglion
2

Dorsal root ganglion
3

Hippocampus
4

Basal portion of pons
5

Hypoglossal nerve
Q/Q(M)-482585 Report a Problem


A male patient experiences painful vesicles on the palate, tongue, and ipsilateral eardrum. Additionally,
he complains of vertigo and difficulty with speaking and eating. Involvement of what anatomic structure
is responsible for this constellation of symptoms?
1

Geniculate ganglion
The question stem describes Ramsay-Hunt syndrome, which is characterized by unilateral vesicles on
palate, tongue, earlobe, and eardrum, in association with neurologic symptoms including vertigo,
difficulty speaking, difficulty eating, hearing loss, and facial droop. The syndrome is caused by
varicella-virus infection of the geniculate ganglion.
Q/Q(M)-482585 Report a Problem

High doses of which of the following antiviral agents has been associated with thrombotic
thrombocytopenic purpura in immunosuppressed patients?
1

Valcyclovir
2

Acyclovir
3

Foscarnet
4

Famciclovir
5

Cidofovir
Q/Q(M)-473922 Report a Problem



79

High doses of which of the following antiviral agents has been associated with thrombotic
thrombocytopenic purpura in immunosuppressed patients?
1

Valcyclovir
Thrombotic thrombocytopenic purpura has been reported with high doses of valcyclovir in
immunosuppressed patients.
Q/Q(M)-473922 Report a Problem


How long is the life cycle for the mite that causes this infestation?
1

1 day
2

1 week
3

1 month
4

6 months
5

1 year
Q/Q(M)-476874 Report a Problem


How long is the life cycle for the mite that causes this infestation?
3

1 month
Scabies is caused by the mite Sarcoptes scabiei var. hominis. The life cycle of the mite is 30 days. A
female mite will lay 60-90 eggs during her life.
Q/Q(M)-476874 Report a Problem


Each of the following is true regarding leishmaniasis except:
1

Arthropod vector is the sand fly
2

Mazzottis test is diagnostic
3

Pentavalent antimony used for visceral disease
4

Cutaneous disease is the most common form
80

5

Espundia seen mucocutaneous disease
Q/Q(M)-477344 Report a Problem

Each of the following is true regarding leishmaniasis except:
2

Mazzottis test is diagnostic
Leishmaniasis is a parasitic infection born by a sandfly vector. No treatment is necessary for cutaneous
leishmaniasis, pentavalent antimony for visceral leishmaniasis. Mazzotti reaction is used to test for
onchocerciasis in which a single dose of dimethycarbamazine is given for reaction.
Q/Q(M)-477344 Report a Problem

Bacillary angiomatosis is caused by which of the following organisms?
1

B. quintana
2

B. henselae
3

B. bacilliformis
4

B. quintana and B. henselae
5

All of these answers are correct
Q/Q(M)-473953 Report a Problem

Bacillary angiomatosis is caused by which of the following organisms?
4

B. quintana and B. henselae
B. quintana or B. henselae are both causative organisms for bacillary angiomatosis.
Q/Q(M)-473953 Report a Problem

A 42 year-old woman presents with a large, vegetating ulcer involving her left labia majora and groin
for over 1 year. A Giemsas stained touch preparation reveals bipolar, safety pin-shaped
intracytoplasmic inclusions. What is the most likely diagnosis?
1

Lymphogranuloma venereum
2

Granuloma Inguinale
3

Chancroid
4

Primary herpes simplex
5

Gonorrhea
Q/Q(M)-474273 Report a Problem


A 42 year-old woman presents with a large, vegetating ulcer involving her left labia majora and groin
for over 1 year. A Giemsas stained touch preparation reveals bipolar, safety pin-shaped
intracytoplasmic inclusions. What is the most likely diagnosis?
2

Granuloma Inguinale
Bipolar, safety pin-shaped intracytoplasmic inclusions on Giemsa stain - known as Donovan bodies - are
81

diagnostic of granuloma inguinale.
Q/Q(M)-474273 Report a Problem


A pool worker develops these painful erythematous nodules with ulceration on his fingers and then his
arms. The antibiotic of choice to treat this infection is:
1

Penicillin
2

Ceftriaxone
3

Minocycline
4

Trimethoprim-sulfamethoxasole
5

Dapsone
Q/Q(M)-476665 Report a Problem


A pool worker develops these painful erythematous nodules with ulceration on his fingers and then his
arms. The antibiotic of choice to treat this infection is:
3

Minocycline
Sporotrichoid spread may be seen in cutaneous leishmaniasis, actinomycosis, atypical mycobacterial
infections, deep fungal infection, and melanoma. Mycobacterium marinum is associated with skin injury
followed by exposure to contaminated water, usually from an aquarium, lake, or pool. The treatment of
choice for M. marinum infections is minocycline.
Q/Q(M)-476665 Report a Problem


A 52-year old male develops a brownish-red plaque on his forehead. The lesion has an "apple jelly"
color on diascopy, and the patient has a positive PPD skin test. What is the most likely mechanism of
disease in this patient?
1

Spread from a distant site
2

Exogenous re-infection
3

Primary inoculation
4

Contiguous spread to skin from underlying infection
5

Id reaction
Q/Q(M)-482275 Report a Problem

A 52-year old male develops a brownish-red plaque on his forehead. The lesion has an "apple jelly"
82

color on diascopy, and the patient has a positive PPD skin test. What is the most likely mechanism of
disease in this patient?
1

Spread from a distant site
This patient has lupus vulgaris, a pauci-bacillary manifestation of tuberculosis infection. Lesions occur
in sensitized patients with a moderate to high immune response and are due to hematogenous,
lymphatic, or contiguous spread from a distant site of infection. Lesions typically occur on the head and
neck. "Apple jelly" color on diascopy is described in lupus vulgaris, but can also be seen in cutaneous
lesions of sarcoidosis.
Q/Q(M)-482275 Report a Problem

The vector of New World Leishmaniasis is the:
1

Phlebotomus sandfly
2

Lutzomyia sandfly
3

Tsteste fly
4

Deer fly (Chrysops spp.)
5

Simulium black fly
Q/Q(M)-473907 Report a Problem


The vector of New World Leishmaniasis is the:
2

Lutzomyia sandfly
The sandfly belonging to the genus Lutzomyia is the vector of New World Leishmaniasis. Sandflies of
the genus Phlebotomus are the vectors of Old World Leishmaniasis.
Q/Q(M)-473907 Report a Problem


Of the choices listed, which antiobiotic is the best to treat a 7-year old child with erythema migrans?
1

Doxycycline
2

Amoxicillin
3

Azithromycin
4

Tetracycline
5

Rifampin
Q/Q(M)-476702 Report a Problem

Of the choices listed, which antiobiotic is the best to treat a 7-year old child with erythema migrans?
2

Amoxicillin
Doxycycline is the usual treatment for erythema migrans in adults in the absence of neurologic or
cardiovascular complications. In children less than 8, all tetracyclines are relatively contraindicated
because of they can cause tooth discoloration. Therefore amoxicillin should be used as the first line
therapy for children less than 8. Note: Changes have been made to the treatment of RMSF. Children of
any age should be treated with doxycycline for that disease.
83

Q/Q(M)-476702 Report a Problem

Verruga peruana is transmitted by:
1

Tick
2

Blackfly
3

Sandfly
4

Fecal oral contact
5

Fecal fecal contact
Q/Q(M)-477426 Report a Problem


Verruga peruana is transmitted by:
3

Sandfly
Verruga peruana, a disease endemic to Peru and a few neighboring countries, is caused by Bartonella
bacilliformis. It is transmitted by the sandfly, Lutzomyia verrucarum. It is preceded by an acute febrile
stage called Oroyo fever or Carrions disease. The treatment of choice is chloramphenicol.
Q/Q(M)-477426 Report a Problem


A patient with HIV/AIDS presents complaining of an asymptomatic patch on the tongue. Examination
reveals a white patch with a corrugated surface on the lateral tongue. What percentage of HIV patients
will have this finding?
1

5%
2

20%
3

33%
4

50%
5

67%
Q/Q(M)-482594 Report a Problem

A patient with HIV/AIDS presents complaining of an asymptomatic patch on the tongue. Examination
reveals a white patch with a corrugated surface on the lateral tongue. What percentage of HIV patients
will have this finding?
3

33%
Oral hairy leukoplakia is characterized by a white patch with a corrugated surface on the side of the
tongue. It is generally asymptomatic. Oral hairy leukoplakia is seen most commonly in patients with
defective cell-mediated immunity, in particular HIV infection. Oral hairy leukoplakia is associated with
EBV opportunisitc infection, among other viruses. 33% of patients with HIV will have oral hairy
leukoplakia.
Q/Q(M)-482594 Report a Problem

A 40-year-old male with HIV (CD4 <200) presents with asymptomatic, slowly enlarging yellow-pink
papules and nodules localized to his perianal skin. A skin biopsy is performed, which reveals foamy
84

dermal interstitial histiocytes with characteristic basophilic laminated inclusion bodies. The latter are
noted to stain positively with von Kossa, Perls, and PAS stains. The most likely diagnosis is:
1

eruptive xanthomas
2

malakoplakia
3

hidradenitits suppurtiva
4

granuloma inguinale
5

chancroid
Q/Q(M)-482835 Report a Problem


A 40-year-old male with HIV (CD4 <200) presents with asymptomatic, slowly enlarging yellow-pink
papules and nodules localized to his perianal skin. A skin biopsy is performed, which reveals foamy
dermal interstitial histiocytes with characteristic basophilic laminated inclusion bodies. The latter are
noted to stain positively with von Kossa, Perls, and PAS stains. The most likely diagnosis is:
2

malakoplakia
Malakoplakia is a multi-organ inflammatory granulomatous disease that most frequently arises in the
setting of immunosuppression. Common sites of involvement include the genitourinary, lung, lymph
nodes and bone. Cutaneous involvement is rare, and typically presents as asymptomatic papules,
nodules, or plaques. Lesions may have associated erosions or ulcerations, and sinus tracts may be seen.
While skin lesions are non-specific, histopathology reveals foamy dermal histiocytes known as von
Hansemann cells which contain basophilic granular inclusions. The latter are known as Michaelis-
Gutmann bodies and are considered pathognomonic for malakoplakia. Michaelis-Gutmann bodies are
comprised of calcium, iron, and phosphate salts, and hence stain positively with von Kossa, Perls, and
PAS, respectively. The pathophysiology of malakoplakia is thought to arise from an acquired defect in
macrophage activation in response to bacterial infection, most commonly. E. coli. Treatment for
cutaneous lesions is surgical excision, although recurrence is not uncommon.
Q/Q(M)-482835 Report a Problem


What is the characteristic body on histopathology for this disease?
1

Cowdry Type A
2

Cowdry Type B
3

Henderson-Patterson
4

Guarnieri
5

Dutcher
Q/Q(M)-476866 Report a Problem
85



What is the characteristic body on histopathology for this disease?
1

Cowdry Type A
Herpes simlex virus is characterized by Cowdry type A bodies on histopathology. They appear as
sharply demarcated eosinophilic structures separated by a clear halo from a basophilic rim of the
infected cell's marginated nuclear chromatin. Cowdry B bodies may be seen in polio infections.
Q/Q(M)-476866 Report a Problem


A child presents with fever, cutaneous tenderness and erythema of flexural and periorifacial areas.
Within 24 hours the erythema progresses to flaccid blisters and erosions. No organisms are cultured and
Nikolsky's sign is positive. What is the most likely diagnosis:
1

Staph scalded skin syndrome
2

Bullous impetigo
3

Fogo selvagem
4

Pemphigus
5

Pemphigoid
Q/Q(M)-481614 Report a Problem


A child presents with fever, cutaneous tenderness and erythema of flexural and periorifacial areas.
Within 24 hours the erythema progresses to flaccid blisters and erosions. No organisms are cultured and
Nikolsky's sign is positive. What is the most likely diagnosis:
1

Staph scalded skin syndrome
Staph scaled skin syndrome is caused by exfoliative toxins A and B (ET-A and ET-B) produced by S.
aureus (phage group II, types 71 and 55 most common). Histopathology of the skin shows superficial
intraepidermal splitting without epidermal necrosis with very few inflammatory cells. Desmoglein 1 is
the specific receptor for exfoliative toxin A cleavage. Desmoglein 1 is the antigen targeted in Fogo
selvagem, the vector for which is the black fly. Bullous impetigo is caused by Staph and is a subgranular
blister with bacteria and neutrophils on histopahtology. Pemphigoid and pemphigus would be less likely
in a child.
Q/Q(M)-481614 Report a Problem


Which of the following is the vector responsible for the transmission of Verruga Peruana?
1

Lutzomyia verrucarum
2

Tsetse fly
86

3

Ctenocephalides felis
4

Pediculus humanus corporis
5

Simulium slossonae
Q/Q(M)-473942 Report a Problem

Which of the following is the vector responsible for the transmission of Verruga Peruana?
1

Lutzomyia verrucarum
Verruga Peruana is caused by infection with B. bacilliformis, which is transmitted by the sandfly,
Lutzomyia verrucarum.
Q/Q(M)-473942 Report a Problem


Contagious pustular dermatitis (Orf) is caused by a:
1

Herpesvirus
2

Poxvirus
3

Gram-positive spore-forming rod
4

Paramyxovirus
5

Papovavirus
Q/Q(M)-474529 Report a Problem

Contagious pustular dermatitis (Orf) is caused by a:
2

Poxvirus
Orf is caused by orf virus (OV), a poxvirus of the genus Parapoxvirus.
Q/Q(M)-474529 Report a Problem

Herpes simplex virus infection and this condition has been linked to which HLA type?
1

HLA Cw6
2

HLA B15
3

HLA B27
4

HLA B51
5

HLA DR3
Q/Q(M)-476753 Report a Problem
87




Herpes simplex virus infection and this condition has been linked to which HLA type?
2

HLA B15
HSV related erythema multiforme has been associated with an increased frequency of HLA-B15.
Q/Q(M)-476753 Report a Problem


A 23 year old female patient is found to have genital warts. A referral to an gynecologist reveals that she
has cervical dysplasia. What viral proteins does HPV use to induce neoplasia?
1

E6 and E7
2

E1 and E2
3

L1 and L2
4

E8 and L1
5

E3 and E4
Q/Q(M)-482610 Report a Problem

A 23 year old female patient is found to have genital warts. A referral to an gynecologist reveals that she
has cervical dysplasia. What viral proteins does HPV use to induce neoplasia?
1

E6 and E7
Certain HPV subtypes have been shown to induce neoplasia, including cervical cancer. There are many
high-risk subtypes, including 16, 18, 31, and 33. The HPV early genes E6 and E7 bind the tumor
suppressors p53 and retinoblastoma protein, respectively, inactivating them and initiating oncogenesis.
Q/Q(M)-482610 Report a Problem
88


A 28 year-old man presents with fever, chills, arthralgias and the cutaneous lesions shown. The most
likely diagnosis is:
1

Purpura fulminans
2

Rickettsialpox
3

Anthrax
4

Gonoccemia
5

Primary herpes simplex infection
Q/Q(M)-474912 Report a Problem



A 28 year-old man presents with fever, chills, arthralgias and the cutaneous lesions shown. The most
likely diagnosis is:
4

Gonoccemia
The image shows sparsely distributed hemorrhagic vesiculopustules with erythematous bases on an acral
surface. These features are characteristic of gonococcemia.
Q/Q(M)-474912 Report a Problem


What percentage of patients undergoing stem cell transplantation for leukemia will develop herpes
zoster within the first year?
1

5%
2

30%
3

50%
4

70%
5

90%
Q/Q(M)-482586 Report a Problem

What percentage of patients undergoing stem cell transplantation for leukemia will develop herpes
zoster within the first year?
89

4

70%
The risk of developing herpes zoster following stem cell transplantation for leukemia is quite high,
approaching 70% in some studies.
Q/Q(M)-482586 Report a Problem

A patient presents with multiple asensate patches on the abdomen. A diagnosis of leprosy is made.
Which of the following regimens is the WHO recommendation for treatment of paucibacillary leprosy?
1

Dapsone and rifampin
2

Dapsone and clofazimine
3

Clofazimine and rifampin
4

Minocycline and rifampin
5

Minocycline and clofazmine
Q/Q(M)-482692 Report a Problem

A patient presents with multiple asensate patches on the abdomen. A diagnosis of leprosy is made.
Which of the following regimens is the WHO recommendation for treatment of paucibacillary leprosy?
1

Dapsone and rifampin
Paucibacillary leprosy is characterized by three or fewer anesthetic and anhidrotic lesions. This form of
leprosy develops when the body mounts a Th1 cytokine profile leading to rare bacilli found in the
lesions. The WHO recommends treatment of paucibacillary leprosy with dapsone 100 mg daily for 6
months and rifampin 600 mg monthly for 6 months.
Q/Q(M)-482692 Report a Problem



Guarneri bodies are associated with:
1

Measles
2

Orf
3

CMV
4

Smallpox
5

Anthrax
Q/Q(M)-474531 Report a Problem

Guarneri bodies are associated with:
4

Smallpox
Guarnieris bodies are cytoplasmic eosinophillic inclusions found on light microscopy.
Q/Q(M)-474531 Report a Problem



90

Winterbottoms sign is a characteristic feature of which of the following diseases?
1

Leishmaniasis
2

Onchocerciasis
3

Loaiasis
4

African Trypanosomiasis
5

Strongyloidiasis
Q/Q(M)-474543 Report a Problem

Winterbottoms sign is a characteristic feature of which of the following diseases?
4

African Trypanosomiasis
Winterbottoms sign (posterior cervical lymphadenopathy) is a clinical feature of African
trypanosomiasis. Romaas sign (eyelid edema and conjunctivitis at site of incoculation) is a
diagnostic finding in American trypanosomiasis (Chagas disease).
Q/Q(M)-474543 Report a Problem


The usual cause ecthyma is:
1

Group A streptococcus
2

Group B streptococcus
3

Group D streptococcus
4

Staphylococcus Aureus
5

Lancefield type O organisms
Q/Q(M)-480157 Report a Problem

The usual cause ecthyma is:
1

Group A streptococcus
The usual cause of ecthyma is Group A streptococcus, though other organisms can occasionally cause
similar cutaneous infections. Group B streptococcus is S. agalactae, commonly isolated from normal
vaginal mucosa. Infections can be problematic for neonates, but not a frequent cause of ecthyma. Group
D streptococcus/Lancefield type O organisms are enterococci. S. aureus can secondarily infect a lesion
of ecthyma but is not the usual cause of this condition.
Q/Q(M)-480157 Report a Problem

What is the most important predictor of materal-fetal transmission of HSV-2?
1

Use of pre-natal antiviral medications
2

Length of delivery
3

Nature of maternal infection
4

Nutrition status of the mother
5

Use of peri-natal monitoring equipment.
91

Q/Q(M)-482575 Report a Problem

What is the most important predictor of materal-fetal transmission of HSV-2?
3

Nature of maternal infection
The nature of the maternal infection (primary versus recurrent) is the most important predictor of
maternal-fetal transmission of HSV-2. In mothers with a primary infection, the risk of transmission is
25-50%. In mothers experiencing a recurrent infection, the risk is 2-5%
Q/Q(M)-482575 Report a Problem

What rare hematologic disorder can accompany varicella-zoster virus infection?
1

Thrombocytopenia
2

Neutropenia
3

Lymphopenia
4

Lymphocytosis
5

Anemia
Q/Q(M)-482578 Report a Problem



What rare hematologic disorder can accompany varicella-zoster virus infection?
1

Thrombocytopenia
Symptomatic thrombocytopenia, while rare, has been associated with acute varicella-zoster virus
infection.
Q/Q(M)-482578 Report a Problem


A laboratory worker undergoes injection of vaccinia virus for vaccination against smallpox. What type
of reaction is needed to ensure adequate development of immunity?
1

Vesicle or ulcer surrounded by 4 cm of erythema
2

Lichenoid plaque
3

Development of vesicles at site separate from injection
4

Targetoid plaque
5

Systemic flu-like reaction with fever and myalgia
Q/Q(M)-482601 Report a Problem


A laboratory worker undergoes injection of vaccinia virus for vaccination against smallpox. What type
of reaction is needed to ensure adequate development of immunity?
1

Vesicle or ulcer surrounded by 4 cm of erythema
Vaccinia virus is used to vaccinate high-risk individuals against smallpox virus. Development of a
vesicle or ulcer with 4 cm of surrounding erythema is a reaction considered to have a high rate of
adequate immunization.
92

Q/Q(M)-482601 Report a Problem

A 25 year old woman who is 13 weeks pregnant is exposed to cytomegalovirus (CMV). Three weeks
later, she develops infectious mononucleosis symptoms. What percentage of neonates with congenital
CMV infection are asymptomatic?
1

10%
2

25%
3

50%
4

75%
5

90%
Q/Q(M)-482593 Report a Problem


A 25 year old woman who is 13 weeks pregnant is exposed to cytomegalovirus (CMV). Three weeks
later, she develops infectious mononucleosis symptoms. What percentage of neonates with congenital
CMV infection are asymptomatic?
5

90%
Symptoms of congenital CMV infection include low birth weight, microcephaly, seizures, petechial
rash, and hepatosplenomegaly. Fortunately, the vast majority (90%) of congenital CMV infections are
asymptomatic.
Q/Q(M)-482593 Report a Problem

A goatherder develops this painless pustule that subsequently forms a black eschar. What causes the
edema to form in this lesion?
1

Increased levels of cAMP
2

Release of TNF-alpha
3

Release of IL-1beta
4

Inhibition of phagocytosis
5

C1 esterase inhibitor
Q/Q(M)-476661 Report a Problem



93

A goatherder develops this painless pustule that subsequently forms a black eschar. What causes the
edema to form in this lesion?
1

Increased levels of cAMP
Anthrax is caused by Bacillus anthracis, a gram positive spore-forming rod. Anthrax is primarily seen in
individuals who are in contact with wild or domestic animals. Recent cases have been linked with
bioterrorism. B. anthracis produces edema toxin and lethal toxin. Edema toxin is comprised of edema
factor and protective antigen. Edema factor is a calmodulin dependent adenyl cyclase. The increased
cAMP induces the gelatinous edema of anthrax skin lesions.
Q/Q(M)-476661 Report a Problem


The causative organism of epidemic typhus is:
1

Francisella tularensis
2

R. prowazekii
3

R. akari
4

R. typhi
5

Salmonella typhi
Q/Q(M)-474245 Report a Problem

The causative organism of epidemic typhus is:
2

R. prowazekii
Epidemic typhus is caused by infection with R. prowazekii.
Q/Q(M)-474245 Report a Problem

The usual culture medium for Mycobacterium leprae is:
1

Chocolate agar in 10% CO2
2

Lowenstein-Jensen
3

It cannot be cultured
4

Sheeps blood agar
5

Agar supplemented with heme and nicotinamide
Q/Q(M)-480178 Report a Problem

The usual culture medium for Mycobacterium leprae is:
3

It cannot be cultured
M. leprae cannot be grown in media or cell culture. It has been grown in mice footpads and in
armadillos. Humans are the only natural host. It is endemic in the tropics and found worldwide. The
Lowenstein-Jensen medium is used to culture mycobacteria most commonly. Neisseria gonorrheae is
grown on chocolate agar in 10% CO2. Agar supplemented with heme (x-factor) and nicotinamide (v-
factor) is needed for Hemophilus influenzae cultures. Sheeps blood agar is useful for identifying
hemolytic strains of streptococcus and staphylococcus.
Q/Q(M)-480178 Report a Problem
94


A 25-year old male from Egypt complains of slowly enlarging chronic growths on his external nares that
are causing nasal obstruction. Biopsy reveals Mikulicz cells on histopathology. Which of the following
medications would be most appropriate for treatment of these lesions?
1

Ciprofloxacin
2

Penicillin
3

Erythromycin
4

Trimethoprim-Sulfamethoxazole
5

Imipenem
Q/Q(M)-482274 Report a Problem


A 25-year old male from Egypt complains of slowly enlarging chronic growths on his external nares that
are causing nasal obstruction. Biopsy reveals Mikulicz cells on histopathology. Which of the following
medications would be most appropriate for treatment of these lesions?
1

Ciprofloxacin
This patient has rhinoscleroma, a chronic granulomatous infection of the nose and upper respiratory tract
by Klebsiella pneumoniae rhinoscleromatis. The disease is endemic in Africa, Southeast Asia, Mexico,
Central and South America, and Central and Eastern Europe and is rarely reported in the United States.
Presentation may vary from rhinitis to diffuse airway obstruction. Mikulicz cells are foamy, vacuolated
macrophages seen in rhinoscleroma. Antibacterial therapeutic choices include ciprofloxacin,
tetracycline, and rifampin. Surgical treatment may be required for obstructive lesions.
Q/Q(M)-482274 Report a Problem

In addition to Kaposis sarcoma, HHV-8 infection is also the causative agent in:
1

Pityriasis rosea
2

Angiosarcoma
3

Primary effusion lymphoma
4

Bacillary angiomatosis
5

Pyogenic granuloma
Q/Q(M)-477224 Report a Problem


In addition to Kaposis sarcoma, HHV-8 infection is also the causative agent in:
3

Primary effusion lymphoma
Human herpes virus 8 is a double stranded DNA virus. It is thought to be pathogenic in Kaposis
sarcoma, primary effusion lymphoma, and Castlemans disease. Primary effusion lymphoma is a rare
B-cell lymphoma seen predominantly in patients with AIDS.
Q/Q(M)-477224 Report a Problem
The main deterrent to staphylococcal infection of the skin is:
1

Complement
95

2

Neutrophils
3

Macrophages
4

Immunoglobulin
5

CD8+ T-cell mediated immunity
Q/Q(M)-480151 Report a Problem

The main deterrent to staphylococcal infection of the skin is:
2

Neutrophils
The main deterrent to staphylococcal infection is the neutrophils. These contain bactericidal chemicals
and proteins such as myeloperoxidase-H202-halide, lysozyem, serine proteases, defensins and bacterial
permeability increasing peptide. Complement serves as an effective opsonin for phagocytosis by
circulating neutrophils and macrophages. Immunoglobulin response hastens phagocytosis but is not
absolutely required. Cytotoxic T-cells do not play a major role in controlling S. aureus infections.
Q/Q(M)-480151 Report a Problem

Epidermodysplasia verruciformis is caused by which human papilloma virus (HPV) types?
1

6 and 11
2

16 and 18
3

5, 8, and 9
4

3 and 10
5

13 and 32
Q/Q(M)-474296 Report a Problem


Epidermodysplasia verruciformis is caused by which human papilloma virus (HPV) types?
3

5, 8, and 9
Epidermodysplasia verruciformis is caused by HPV types 5, 8, 9, 12, 14, 15, 17, 19-26.
Q/Q(M)-474296 Report a Problem

A 24 year-old man from New York City develops fever, chills, malaise, and a severe headache. His
apartment has a mouse problem. On examination, multiple, discrete papulovesicles and a single eschar
are found. What is the most likely diagnosis?
1

Ehrlichiosis
2

Rickettsialpox
3

Q Fever
4

Meningococcemia
5

Rocky Mountain Spotted Fever
Q/Q(M)-474271 Report a Problem
96




A 24 year-old man from New York City develops fever, chills, malaise, and a severe headache. His
apartment has a mouse problem. On examination, multiple, discrete papulovesicles and a single eschar
are found. What is the most likely diagnosis?
2

Rickettsialpox
The clinical description is that of rickettsialpox, which is caused by R. akari and spread by liponyssoides
sanguineus - the house mouse mite.
Q/Q(M)-474271 Report a Problem


Which of the following is the causative organism of Tularemia?
1

Burkholderia mallei
2

Streptobacillus moniliformis
3

Francisella tularensis
4

Orientia tsutsugamushi
5

Klebsiella pneumoniae
Q/Q(M)-474268 Report a Problem


Which of the following is the causative organism of Tularemia?
3

Francisella tularensis
Tularemia is caused by infection with Francisella tularensis - most commonly acquired after contact
with infected rabbits (e.g. hunters).
Q/Q(M)-474268 Report a Problem


Which of the following is the vector of lymphatic filariasis caused by Wuchereria bancrofti?
1

Black fly
2

Tsteste fly
3

Mosquito
4

Deerfly
5

Botfly
Q/Q(M)-474542 Report a Problem


Which of the following is the vector of lymphatic filariasis caused by Wuchereria bancrofti?
3

Mosquito
Lymphatic filariasis is spread by mosquitoes belonging to genera Aedes, Anopheles, Culex, or
Mansonia.
Q/Q(M)-474542 Report a Problem

What is the classic CXR finding of a patient with inhalational anthrax?
97

1

Alveolar infiltrates
2

Increased interstitial markings
3

Pleural effusions
4

Widened mediastinum
5

Normal x-ray
Q/Q(M)-477506 Report a Problem



What is the definition of disseminated herpes zoster?
1

Involvement of two or more defined dermatomes
2

Symmetric bilateral involvement
3

Involvement of one cranial and one spinal nerve.
4

More than 20 vesicles outside of the primarily affected dermatome
5

Involvement of the trunk and at least one extremity.
Q/Q(M)-482582 Report a Problem


What is the definition of disseminated herpes zoster?
4

More than 20 vesicles outside of the primarily affected dermatome
The definition of dissemianted herpes zoster is at least 20 vesicles seen outside of the primarily affected
dermatome. The other option choices are distractors.
Q/Q(M)-482582 Report a Problem


An HIV infected male, presents with a two-week history of a painless 2 cm ulcer on the penile shaft,
with associated inguinal lymphadenopathy. What is the best treatment?
1

Valacyclovir
2

Benzathine penicillin G
3

Ceftriaxone
4

Doxycycline
5

Azithromycin
Q/Q(M)-482813 Report a Problem


An HIV infected male, presents with a two-week history of a painless 2 cm ulcer on the penile shaft,
with associated inguinal lymphadenopathy. What is the best treatment?
2

Benzathine penicillin G
A painless ulcer in an HIV patient is most likely a syphilitic chancre. The treatment of choice is
benzathine penicillin G in patients without a penicillin allergy. Alternative agents in allergic patients
include doxycycline, tetracycline, erythromycin, and ceftriaxone. Valacyclovir would be the treatment
of choice for genital herpes, which is typically painful. Azithromycin in a single 1 g oral dose is
98

effective for chancroid, which is painful.
Q/Q(M)-482813 Report a Problem


What is the first line treatment of streptococcal toxic shock syndrome?
1

Penicillin
2

Ciprofloxacin
3

Clindamycin
4

Doxycycline
5

Vancomycin
Q/Q(M)-482852 Report a Problem


What is the first line treatment of streptococcal toxic shock syndrome?
3

Clindamycin
Clindamycin is considered 1st line treatment of streptococcal toxic shock syndrome, as it inhibits
secretion of the exotoxin.
Q/Q(M)-482852 Report a Problem



Which of the following mycobacterial organisms is classified as a photochromogen?
1

M. Kansasii
2

M. grodonae
3

M. fortuitum
4

M. ulcerans
5

M. tuberculosis
Q/Q(M)-477652 Report a Problem


Which of the following mycobacterial organisms is classified as a photochromogen?
1

M. Kansasii
M. Kansasii, M. marinum, and M. simiae are photochromogens. They form pigment when exposed to
light.
Q/Q(M)-477652 Report a Problem


A 4 year old girl experiences systemic symptoms including fever, fatigue, headache, and vomiting. An
associated exanthem consists of a non-pruritic eruption including the formation of blisters on the palms
and soles and an intra-oral ulcer. Hand-foot-mouth syndrome is diagnosed. What nail finding can be
seen approximately 1 month after infection with Coxsackie virus and other Enteroviruses?
1

Onychomadesis
2

Onycholysis
99

3

Trachyonychia
4

Terry\'s nails
5

Paronychia
Q/Q(M)-482606 Report a Problem


A 4 year old girl experiences systemic symptoms including fever, fatigue, headache, and vomiting. An
associated exanthem consists of a non-pruritic eruption including the formation of blisters on the palms
and soles and an intra-oral ulcer. Hand-foot-mouth syndrome is diagnosed. What nail finding can be
seen approximately 1 month after infection with Coxsackie virus and other Enteroviruses?
1

Onychomadesis
Although rare, onychomadesis has been associated with enteroviral infections, including hand-foot-
mouth syndrome due to Coxsackie virus. Onychomadesis is an idiopathic shedding of the nails
beginning at the proximal end. It may be due to temporary arrest of the matrix during the infection. In
most cases, the nails return normally.
Q/Q(M)-482606 Report a Problem


Each of the following species may be involved in necrotizing fasciitis except:
1

Pseudomonas
2

Clostridium
3

Streptococcus
4

Mycobacterium
5

Bacteroides
Q/Q(M)-477223 Report a Problem

Each of the following species may be involved in necrotizing fasciitis except:
4

Mycobacterium
Necrotizing fasciitis is a rapidly advancing acute necrotizing infection that may follow trauma, surgery,
or occur spontaneously. It is associated with systemic toxicity and high mortality rate. Clinically,
erythema, edema, and edema progresses to dusky cyanosis, blistering and necrosis. Many virulent
bacteria have been culturesd including Pseudomonas, Bacteroides, streptococci, staphylococcus,
enterococci, and clostridium. Both aerobic and anaerobic cultures should be taken. Mycobacterium is
not associated with necrotizing fasciitis.
Q/Q(M)-477223 Report a Problem

WHO recommendations for the treatment of multibacillary leprosy include all of the following
EXCEPT:
1

Dapsone
2

Clofazimine
3

Clarithromycin
4

Rifampin
100

5

All of these answers are recommended for the treatment of multibacillary leprosy.
Q/Q(M)-473937 Report a Problem


WHO recommendations for the treatment of multibacillary leprosy include all of the following
EXCEPT:
3

Clarithromycin
Clarithromycin is not part of the WHO recommendations for the treatment of multibacillary leprosy.
Q/Q(M)-473937 Report a Problem


Which human papillo viurs (HPV) type is implicated in papillomatosis cutis carcinoides di Gottron?
1

7
2

11
3

18
4

57
5

60
Q/Q(M)-477636 Report a Problem


Which human papillo viurs (HPV) type is implicated in papillomatosis cutis carcinoides di Gottron?
2

11
Papillomatosis cutis carcinoides di Gottron, also know as Gottron's tumor, is a verrucous carcinoma of
the skin. In 1948, Ackerman first described verrucous carcinoma (VC), a low-grade tumor that generally
is considered a clinicopathologic variant of squamous cell carcinoma. Verrucous carcinomas are caused
by HPV types 6 and 11. When they occur on the feet they are called epithelioma caniculatum. When
they occur on the genitals they are refered to as Buschke-Lowenstein tumors. When they occur in the
mouth they are called Ackerman's tumors or oral florid papillomatosis.
Q/Q(M)-477636 Report a Problem

Which of the following tests is the most sensitive serologic test in primary syphilis?
1

FTA-ABS
2

VDRL
3

ELISA
4

RPR
5

MHA-TP
Q/Q(M)-482696 Report a Problem


Which of the following tests is the most sensitive serologic test in primary syphilis?
1

FTA-ABS
The fluorescent treponemal antibody absorption test is the most sensitive serologic test in primary
101

syphilis. The ELISA is the test of choice in early primary and congenital syphilis because it is the first to
become reactive. However, it is less sensitive in late primary syphilis because of reduction of IgM
production. The ELISA is 100% specific. MHA-TP is less sensitive than FTA-ABS during primary
syphilis.
Q/Q(M)-482696 Report a Problem



The treatment of choice of the tropical infectious disease shown is:
1

Ivermectin
2

Permethrin
3

Diethylcarbamazine
4

Pentavalent antimony
5

Mebendazole
Q/Q(M)-476070 Report a Problem



The treatment of choice of the tropical infectious disease shown is:
4

Pentavalent antimony
The ulcerated nodule shown here is characteristic of cutaneous leishmaniasis (CL). Leishmaniasis is
treated with pentavalent antimony.
Q/Q(M)-476070 Report a Problem



A patient on the bone marrow transplant service has a fever, neutropenia, hemorrhagic bullae and
erythematous nodules with dusky gray centers. The organism most likely to be responsible is:
1

Streptococcus pyogenes
2

Mycobacterium tuberculosis
3

Candida albicans
4

Pseudomonas aeruginosa
5

Staphylococcus aureus
Q/Q(M)-477315 Report a Problem


A patient on the bone marrow transplant service has a fever, neutropenia, hemorrhagic bullae and
erythematous nodules with dusky gray centers. The organism most likely to be responsible is:
4

Pseudomonas aeruginosa
102

Ecthyma gangrenosum is an infection of critically ill or immunocompromised individuals by
Pseudomonas aeruginosa. Clinically, patients develop opalescent, tense vesicles or pustules surrounded
by a narrow pink to violaceous halo. The lesions rapidly become hemorrhagic, then violaceous and
necrotic leaving ulcers. The most common location is on the buttocks and lower extremities. Treatment
is with aminoglycosides.
Q/Q(M)-477315 Report a Problem

An otherwise healthy six-day-old boy is brought into clinic by his mother with the complaint of tiny
thin-roofed vesicles with a rim of inflammation and a few lesions covered with a honey-colored
crust. Which organism is the most likely cause in this newborn?
2

Group B streptococcus
Group B streptococcus is S. agalactae, commonly isolated from normal vaginal mucosa. In neonates,
impetigo caused by group B streptococci may develop. Listeria monocytogenes is a small, gram-positive
organism that may present as petechial, papular or pustular eruptions in acutely ill infants, usually those
that were meconium stained at birth. The usual cause of ecthyma is Group A streptococcus, though other
organisms can occasionally cause similar cutaneous infections. Group D streptococcus/Lancefield type
O organisms are enterococci. S. aureus can secondarily infect a lesion of ecthyma but is not the usual
cause of this condition.
Q/Q(M)-480158 Report a Problem


A 10 year-old girl presents to the dermatology clinic with a pruritic eruption on her trunk after
swimming at a beach in Florida. Of note, the lesions are limited to skin that was covered by her bathing
suit. The most likely diagnosis is:
1

Seabather?s eruption
2

Swimmer?s itch
3

Cercarial dermatitis
4

Swimming pool granuloma
5

Cutaneous larva migrans
Q/Q(M)-474532 Report a Problem


A 10 year-old girl presents to the dermatology clinic with a pruritic eruption on her trunk after
swimming at a beach in Florida. Of note, the lesions are limited to skin that was covered by her bathing
suit. The most likely diagnosis is:
1

Seabather?s eruption
Seabather?s eruption characteristically involves areas covered by swimwear, as a result of coelenterate
larvae becoming trapped underneath bathing suits. The causative organisms are larval forms of
Edwardsiella lineata (sea anemone) and Linuche unquiculata (thimble jellyfish).
Q/Q(M)-474532 Report a Problem


For which of the following smallpox vaccination complications is the administration of vaccine immune
globulin indicated?
1

Post-vaccinal encephalitis
103

2

Erythema multiforme
3

Eczema vaccinatum
4

Vaccinia keratitis
5

Mild generalized vaccinia
Q/Q(M)-477373 Report a Problem

For which of the following smallpox vaccination complications is the administration of vaccine immune
globulin indicated?
3

Eczema vaccinatum
Vaccinia immune globulin can be administered to treat some of the complications associated with
vaccinia vaccination. Vaccinia immune globulin is indicated for inadvertent inoculation with extensive
involvement or ocular implantations, eczema vaccinatum, generalized vaccinia (severe or recurrent), and
progressive vaccinia. It is not recommended for inadvertent inoculation with mild reaction, generalized
vaccinia (mild or limited), post-vaccination encephalitis, and isolated vaccinia keratitis.
Q/Q(M)-477373 Report a Problem


A 35 year old male has had frequent sex with prostitutes. He presents complaining of the rapid onset of a
new rash on his legs. A review of symptoms is otherwise unremarkable. Examination reveals palpable
purpura and tender nodules of the lower legs. Biopsy reveals vasculitis of small- and medium-sized
vessels. What percentage of patients with this disease are found to be infected with Hepatitis B virus?
1

7-8%
Polyarteritis nodosa is characterized by the onset of painful nodules on the lower extremities. It can be
limited to the skin (cutaneous PAN), or can involve a wide variety of organ systems (systemic PAN).
Approximately 7-8% of cases of polyarteritis nodosa cases are associated with Hepatitis B virus
infection.
Q/Q(M)-482596 Report a Problem

A 48-year-old Latin American female with a diagnosis of leprosy develops bullous lesions that appear
rapidly and ulcerate, especially below the knee. This most likely represents:
1

Reversal reaction
2

Upgrading reaction
3

Erythema nodosum leprosum
4

Lucio's phenomenon
5

Lofgren's syndrome
Q/Q(M)-482352 Report a Problem


A 48-year-old Latin American female with a diagnosis of leprosy develops bullous lesions that appear
rapidly and ulcerate, especially below the knee. This most likely represents:
4

Lucio's phenomenon
This patient has a type III reactional state to leprosy called Lucio's phenomenon. It mostly occurs in
104

people of Latin American descent and is characterized by rapidly ulcerating and bullous lesions usually
on the legs. It has a high mortality rate. A type I reaction, or a reversal/upgrading reaction, can occur
after treatment and are associated with a change in cell-mediated immunity and result in accelerated
destruction of bacilli. This is manifested as inflammation of existing lesions with no systemic symptoms.
A type II reaction, or erythema nodosum leprosum, represents an immune complex deposition and
features widespread erythematous nodules with systemic features. Lofgren's is associated with
sarcoidosis.
Q/Q(M)-482352 Report a Problem


One of your colleagues returned from a vacation to India with fever, vomiting, pleural effusions, ascites
and conjunctival petechiae. She also has a diffuse macular rash with notable areas of sparing that your
attending refers to as white islands in a sea of red. What is your diagnosis?
1

Leishmania
2

Dengue hemorrhagic fever
3

Malaria
4

Typhoid
5

Leptospirosis
Q/Q(M)-477139 Report a Problem

One of your colleagues returned from a vacation to India with fever, vomiting, pleural effusions, ascites
and conjunctival petechiae. She also has a diffuse macular rash with notable areas of sparing that your
attending refers to as white islands in a sea of red. What is your diagnosis?
2

Dengue hemorrhagic fever
Dengue fever is caused by an Arbovirus and transmitted by the mosquito, Aedes aegypti and may cause
Dengue Shock Syndrome and Dengue Hemorrhagic Fever. The infection starts with sudden onset of
high fever, backache, retroorbital pain, bone and joint pain, depression and malaise. The disease is also
called "break-bone fever." One to seven days after onset of fever, rash presents characteristically starting
on the dorsum and hands and feet spreading to limbs and torso. The eruption may become confluent
with small, round islands of sparing, the so called "white islands in a sea of red." Treatment is generally
supportive as no antivirals are effective.
Q/Q(M)-477139 Report a Problem


The vector of Trench Fever is the:
1

Human body louse (Pediculus humanus corporis)
2

Cat flea (Ctenocephalides felis)
3

Sandfly (Phlebotamus perniciosus)
4

Rat flea (Xenopsylla cheopis)
5

Trombiculid mite
Q/Q(M)-473909 Report a Problem


The vector of Trench Fever is the:
105

1

Human body louse (Pediculus humanus corporis)
The human body louse (Pediculus humanus corporis)is the vector of Trench Fever, Epidemic Typhus,
and Relapsing Fever.
Q/Q(M)-473909 Report a Problem


Which of the following are inconsistent with the diagnosis of staphylococcal scalded skin syndrome?
1

Epidermal changes are produced by exfoliative toxins of staphlococcus
2

Initial event is usually a localized staph infection
3

Swabs and cultures of fluid filled blisters overwhelmingly grow staph
4

Prognosis is good in children with low mortality when anitbiotics are administered early
5

Cell necrosis does not occur in staphylococcal scalded skin syndrome as it does in T.E.N
Q/Q(M)-478752 Report a Problem



Which of the following are inconsistent with the diagnosis of staphylococcal scalded skin syndrome?
3

Swabs and cultures of fluid filled blisters overwhelmingly grow staph
Staphylococcal scalded skin syndrome: Lesions extend far beyond areas of actual staphylococcal
infection, by action of the epidermolytic exotoxin elaborated by the staphylococcus in remote sites.
Usually the staphylococci are present at a distant focus such as the parynx, nose ear, or conjuctiva. If
cultures are taken they should be obtained fromthe mucous membranes because the skin erythema and
desquamation is due to the distant effects of the exfoliative toxin, unlike the sitaution in bullous
impetigo, where S. aureus is present in the lesions. Epidermal changes are produced by exfoliative
toxins of Staphylococcus. Inital event is usually a localized Staph infection. Prognosis is good in
children with low mortality when anitbiotics are administered earyl. Cell necrosis does not occur in SSS
as it does in toxic epidermal necrolysis.
Q/Q(M)-478752 Report a Problem

A patient with HIV/AIDS presents with vesicles in a disseminated distribution. A biopsy is performed.
Which histologic finding favors infection with varicella-zoster virus over herpes simplex virus?
1

Pseudoepitheliomatous hyperplasia
2

Margination of keratinocyte nuclei
3

Multinucleated cells
4

Superficial dermal leukocytoclastic vasculitis
5

Molding of nuclei
Q/Q(M)-482587 Report a Problem


A patient with HIV/AIDS presents with vesicles in a disseminated distribution. A biopsy is performed.
Which histologic finding favors infection with varicella-zoster virus over herpes simplex virus?
4

Superficial dermal leukocytoclastic vasculitis
Herpes simplex virus and disseminated varicella-zoster virus infection can be difficult to distinguish
106

clinically and histologically. These viruses share the histologic findings of margination of nuclei,
multinucleated cells, and nuclear molding. The presence of leukocytoclastic vasculitis favors a diagnosis
of disseminated varicella-zoster virus.
Q/Q(M)-482587 Report a Problem


Which of the extracellular enzymes of S. aureus plays a role in toxic shock syndrome (TSS)?
1

Coagulase
2

Penicillinase
3

Enterotoxin
4

Hemolysin
5

Exfoliatin
Q/Q(M)-480152 Report a Problem


Which of the extracellular enzymes of S. aureus plays a role in toxic shock syndrome (TSS)?
3

Enterotoxin
In addition to TSST-1 which is present in 50% of non-menstrual cases of TSS, enterotoxins, especially
B & C, cause TSS. These enterotoxins are also superantigens, which recognize the V-beta region of the
T-cell receptor. Coagulase clots plasma, Penicillinase degrades penicillin, Hemolysin lyses red blood
cells and exfoliatin splits the epidermis (antigen is desmoglein 1).
Q/Q(M)-480152 Report a Problem

What is the risk of transmission of HSV from active cervical lesions when a C-section is performed?
1

1%
2

8%
3

17%
4

31%
5

43%
Q/Q(M)-482576 Report a Problem


What is the risk of transmission of HSV from active cervical lesions when a C-section is performed?
1

1%
When a C-section is performed in the setting of active cervical lesions of HSV, the risk of maternal-fetal
transmission is decreased to 1%. If a child were delivered vaginally in such a setting, the risk of
transmission is approximately 8%.
Q/Q(M)-482576 Report a Problem


A 16 year old female presents with a sore throat, painful cervical lymphadenopathy, and an enlarged
spleen. An empiric trial of amoxicillin leads to the development of a diffuse morbilliform rash. What is
the incubation period of the most likely viral cause of this patient\'s symptoms?
107

1

3-7 days
2

7-10 days
3

2-3 weeks
4

3-7 weeks
5

6 months
Q/Q(M)-482589 Report a Problem



A 16 year old female presents with a sore throat, painful cervical lymphadenopathy, and an enlarged
spleen. An empiric trial of amoxicillin leads to the development of a diffuse morbilliform rash. What is
the incubation period of the most likely viral cause of this patient\'s symptoms?
4

3-7 weeks
Infectious mononucleosis is characterized by a sore throat, painful lymphadenopathy, fatigue, and
splenomegaly. The formation of a morbilliform rash upon challenge with ampicillin or related
antibiotics is a classic finding. The most common virus associated with infectious mononucleosis is
Epstein-Barr virus. The incubation period is relatively long, 3-7 weeks.
Q/Q(M)-482589 Report a Problem


A 10-year-old boy develops an acute illness and rash along with marked enlargement of the posterior
cervical lymph nodes. This presentation is most consistent with:
1

West African sleeping sickness
2

East African sleeping sickness
3

Chagas disease
4

Schistosomiasis
5

Sparaganosis
Q/Q(M)-482313 Report a Problem

A 10-year-old boy develops an acute illness and rash along with marked enlargement of the posterior
cervical lymph nodes. This presentation is most consistent with:
2

East African sleeping sickness
East African sleeping sickness is caused by Trypanosoma rhodisiense and leads to acute illness with
rapid fatal course and pronounced posterior cervical lymphadenopathy (Winterbottom's sign). West
African sleeping sickness is more chronic. American trypanosomiasis, or Chagas disease, features
unilateral conjunctivitis and edema of the face (Romana's sign) and heart and gastrointestinal sequelae.
Schistomsomiasis causes swimmer's itch, while sparaganosis from ingestion of Spirometra leads to
painful edematous lumps.
Q/Q(M)-482313 Report a Problem

A 24 year-old man from New York City develops fever, chills, malaise, and a severe headache. His
apartment has a mouse problem. On examination, multiple, discrete papulovesicles and a single eschar
are found. What is the most likely diagnosis?
108

1

Ehrlichiosis
2

Rickettsialpox
3

Q Fever
4

Meningococcemia
5

Rocky Mountain Spotted Fever
Q/Q(M)-474271 Report a Problem


A 24 year-old man from New York City develops fever, chills, malaise, and a severe headache. His
apartment has a mouse problem. On examination, multiple, discrete papulovesicles and a single eschar
are found. What is the most likely diagnosis?
2

Rickettsialpox
The clinical description is that of rickettsialpox, which is caused by R. akari and spread by liponyssoides
sanguineus - the house mouse mite.
Q/Q(M)-474271 Report a Problem


What is the most common site of infection from Streptococcus Iniae?
1

Lower leg
2

Face
3

Hand
4

Foot
5

Nails
Q/Q(M)-477612 Report a Problem

What is the most common site of infection from Streptococcus Iniae?
3

Hand
Streptococcus iniae most commonly causes bacteremic cellulitis of the hand in persons who have
recently handled fresh fish. S. iniae is a fish pathogen that causes sporadic infection in tilapia, yellowtail,
rainbow trout, and coho salmon. Treatment of choice is with penicillin antibiotics.
Q/Q(M)-477612 Report a Problem


A 32 year-old woman presents with meningitis and palpable purpura. A diagnosis of meningococcemia
is confirmed via culture of cerebrospinal fluid. Which complement components are most likely to be
deficient in this patient?
1

C1
2

C2
3

C3
4

C4
5

C5
109

Q/Q(M)-477609 Report a Problem

A 32 year-old woman presents with meningitis and palpable purpura. A diagnosis of meningococcemia
is confirmed via culture of cerebrospinal fluid. Which complement components are most likely to be
deficient in this patient?
5

C5
Patients with meningococcemia often demonstrate deficiencies of late complement components C5-9.
Q/Q(M)-477609 Report a Problem

A 24 year old male presents with a high fever, arthralgias, and a rash characterized by nonspecific
erythematous macules in a generalized distribution. He was recently cleaning his family\'s barn, and was
bitten more than once by rats and mice. What is the treatment of choice for this infection?
1

Penicillin
2

Erythromycin
3

Clindamycin
4

Ciprofloxacin
5

Amikacin
Q/Q(M)-482688 Report a Problem


A 24 year old male presents with a high fever, arthralgias, and a rash characterized by nonspecific
erythematous macules in a generalized distribution. He was recently cleaning his family\'s barn, and was
bitten more than once by rats and mice. What is the treatment of choice for this infection?
1

Penicillin
Rat-bite fever is also called Haverhill Fever. It is caused by Streptobacillus moniliformis and is acquired
from infected rodents or by touching or eating contaminated food. The signs and symptoms are
nonspecific, but the diagnosis can be suspected based on history. Treatment is with penicillin.
Q/Q(M)-482688 Report a Problem


A patient presents with painful vesicles in a dermatomal distribution on his forehead and on the tip of his
nose. She is sent by her ophthalmologist who diagnostic ophthalmic zoster. Which nerve branch is
involved?
1

Ophthalmic
2

Nasal
3

Infraorbital
4

Supraorbital
5

Temporal
Q/Q(M)-480181 Report a Problem



110

A patient presents with painful vesicles in a dermatomal distribution on his forehead and on the tip of his
nose. She is sent by her ophthalmologist who diagnostic ophthalmic zoster. Which nerve branch is
involved?
1

Ophthalmic
The ophthalmic branch of the trigeminal branch is involved in ophthalmic zoster. This presentation
accounts for 10-15% of all cases of VZV. The other nerve branches listed are not involved in ophthalmic
zoster.
Q/Q(M)-480181 Report a Problem


A 20-month old child develops a high fever followed 2 days later by a sudden eruption of rose pink
macules and papules with white halos as the fever subsides. What is the most likely diagnosis?
1

Measles
2

Rubella
3

Scarlet fever
4

Exanthem Subitum
5

Erythema infectiosum
Q/Q(M)-474238 Report a Problem


A 20-month old child develops a high fever followed 2 days later by a sudden eruption of rose pink
macules and papules with white halos as the fever subsides. What is the most likely diagnosis?
4

Exanthem Subitum
The cutaneous features of Exanthem Subitum (Roseola infantum, Sixth Disease)as described in the
question develop as the fever subsides. While the other diagnoses are part of the differential diagnosis of
"fever and rash" in children, only roseola infantum has this characteristic clinical course.
Q/Q(M)-474238 Report a Problem


What is the treatment for syphillis in a penacillin allergic patient?
1

Penicillin
2

Doxcycyline
3

Rifampiin
4

Clindamycin
5

Ceftriaxone
Q/Q(M)-476872 Report a Problem

What is the treatment for syphillis in a penacillin allergic patient?
2

Doxcycyline
Secondary syphillis is caused by the spirochete, T. pallidum. Penicillin is the treatment of choice for
secondary syphillis. In penicillin allergic individuals, doxycyline may be used.
Q/Q(M)-476872 Report a Problem
111

También podría gustarte